CME qstns Set 2

Ace your homework & exams now with Quizwiz!

Acute uncomplicated cystitis responds well to 3 days of trimethoprim/sulfamethoxazole, 160/800 mg twice daily. Increasing resistance to fluoroquinolones has been seen and they are therefore less likely to be successful. Because of the association with tendon rupture they are also not considered first-line treatment. β-Lactam agents have similar resistance issues. Azithromycin is not indicated for urinary tract infections.

A 46-year-old female has a 3-day history of dysuria with burning, frequency, and urgency. She reports no fever, weakness, or hematuria. Her chronic health problems include obesity and prediabetes. She has no known allergies. Which one of the following would be the most appropriate treatment regimen for this patient? (check one) A. Ibuprofen for 3 days B. Trimethoprim/sulfamethoxazole (Bactrim) for 3 days C. Ciprofloxacin (Cipro) for 3 days D. Levofloxacin (Levaquin) for 7 days E. Azithromycin (Zithromax) for 7 days

A 90-year-old female with severe dementia is seen in the emergency department for a left knee strain. She was at home alone for 2 hours and no fall or injury was witnessed. A radiograph of the knee is negative, and she is referred to you for follow-up the next day. At the follow-up visit the patient is confused and agitated, and cries out at any attempt to examine her. She is unable to bear weight on her left leg and it appears to be externally rotated. Which one of the following would be most appropriate at this point? (check one) A. Reassurance and pain medication B. A repeat radiograph of the left knee C. A radiograph of the left hip D. A radiograph of the lumbosacral spine E. MRI of the left knee

A fractured hip is possible and must be ruled out since there is difficulty bearing weight and the leg is externally rotated. Examination of a patient with severe dementia can be extremely difficult. Other findings with a fractured hip would include pain elicited on rotation and groin pain when applying an axial load. If the hip radiograph is negative, MRI of the knee may be considered.

A 29-year-old mother of three young children asks your opinion on giving probiotics to her children. Which one of the following is a benefit of the use of probiotics in children? (check one) A. They reduce diarrhea associated with irritable bowl syndrome B. They reduce colic symptoms in formula-fed infants C. They prevent the development of allergies D. They prevent antibiotic-associated diarrhea

A wealth of evidence-based research has established the benefits of probiotics, especially in children. Probiotics, particularly Saccharomyces boulardii , have been shown to prevent the antibiotic-associated diarrhea that occurs in 5%-30% of children who receive antibiotics (SOR A). The number needed to treat to prevent one case of diarrhea is 10. Probiotics reduce the pain associated with irritable bowel syndrome (IBS) but have not been shown to be helpful in reducing diarrhea or constipation in pediatric IBS patients (SOR A). In breastfed infants, probiotics reduce daily crying time by up to an hour. Similar benefits have not been found in formula-fed infants or infants who are combining breastfeeding and formula. Probiotics have not been shown to prevent colic in any infants. Other benefits of probiotics include the prevention of eczema and upper respiratory infections (SOR A). However, their use has not been shown to prevent allergies and asthma. Ref: Johnston BC, Goldenberg JZ, Parkin PC: Probiotics and the prevention of antibiotic-associated diarrhea in infants and children. JAMA 2016;316(14):1484-1485. 2) Islam SU: Clinical uses of probiotics. Medicine (Baltimore) 2016;95(5):e2658. 3) Dassow P, Fox S: When can infants and children benefit from probiotics? J Fam Pract 2016;65(11):789-794.

A 46-year-old female with bipolar I disorder in remission seeks your advice regarding discontinuation of her medications, which include sertraline (Zoloft), 50 mg daily; quetiapine (Seroquel), 200 mg nightly; and hydroxyzine, 25 mg taken occasionally as needed for anxiety. Which one of the following would you advise this patient? (check one) A. Continue all current medications without change B. Discontinue all medications except for as-needed use of hydroxyzine C. Discontinue only quetiapine D. Discontinue either sertraline or quetiapine E. Discontinue quetiapine and start an alternative mood stabilizer

Accepted mood stabilizers used for maintenance therapy in patients with bipolar disorder include lithium, valproate, lamotrigine, and some atypical antipsychotics such as olanzapine, quetiapine, and risperidone. The atypical antipsychotics are associated with weight gain and adverse metabolic changes. Annual testing for diabetes mellitus is recommended. Long-term maintenance therapy with a mood stabilizer is recommended in patients with bipolar I disorder due to the high risk of recurrent mania. Monotherapy with antidepressants is contraindicated. Although the typical antipsychotics may cause QT prolongation, atypical antipsychotics such as quetiapine do so much less frequently, and most SSRIs do not cause this. Co-administration with sertraline is not contraindicated.

A 24-year-old asymptomatic female has a chest radiograph that incidentally shows bilateral hilar adenopathy. Additional evaluation supports a diagnosis of sarcoidosis. Which one of the following would be most appropriate at this point? (check one) A. Monitoring only B. Treatment with corticosteroids C. Treatment with methotrexate D. A transbronchial lung biopsy

According to an international consensus statement, there are three criteria for diagnosing sarcoidosis: (1) a compatible clinical and radiologic presentation, (2) pathologic evidence of noncaseating granulomas, and (3) exclusion of other diseases with similar findings. The main exceptions to the need for histologic confirmation are the presence of bilateral hilar adenopathy in an asymptomatic patient (stage I) and the presentation of sarcoid-specific Lofgren syndrome—with fever, erythema nodosum, and bilateral hilar adenopathy that can be diagnosed based on clinical presentation alone. An asymptomatic patient with stage I sarcoidosis (bilateral hilar lymphadenopathy on chest radiography) without suspected infection or malignancy does not require an invasive tissue biopsy because the results would not affect the recommended management, which is monitoring only. Treatment is not indicated because spontaneous resolution of stage I sarcoidosis is common. Reliable biomarkers for diagnosing sarcoidosis do not exist. Although the serum angiotensin converting enzyme level may be elevated in up to 75% of untreated patients, this lacks sufficient specificity, has large interindividual variability, and fails to consistently correlate with disease severity, all of which limit its clinical utility. Pathologic evidence of noncaseating granulomas from the most accessible and safest biopsy site should be pursued only if there is an indication for treatment, such as significant symptomatic or progressive stage II or III pulmonary disease or serious extrapulmonary disease. If treatment is indicated, corticosteroids are the first-line treatment for sarcoidosis. Second- and third-line treatments include methotrexate, azathioprine, leflunomide, and biologic agents.

A 45-year-old female presents with a 6-month history of fatigue and arthralgias. When asked about recent illnesses the only thing she can recall is that she developed a rash on her face after a picnic about 1 month ago. An examination reveals swelling and tenderness in her left knee and over the proximal interphalangeal joints of the second and third digits of her right hand. An antinuclear antibody test and an anti-double-stranded DNA test are both positive. A CBC and blood chemistries are within normal limits.Which one of the following medications would be the best initial treatment for this patient? (check one) A. Azathioprine (Imuran) B. Cyclosporine C. Hydroxychloroquine (Plaquenil) D. Mycophenolate (CellCept) E. Rituximab (Rituxan)

According to the American College of Rheumatology criteria, this patient has systemic lupus erythematosus, with photosensitivity, arthritis, a positive antinuclear antibody test, and a positive anti-double-stranded DNA test. She has a mild form of the disease. Hydroxychloroquine reduces arthritis pain in lupus patients (SOR A) and is the preferred initial treatment for lupus arthritis. Cyclosporine and azathioprine are indicated for severe lupus or lupus nephritis. Mycophenolate is indicated for refractory lupus or lupus nephritis. Rituximab is indicated for severe refractory lupus.

You are discharging a mother and her infant after delivery. The infant was born at 35 weeks gestation, is exclusively breastfed, and has a weight that is appropriate for her gestational age. No blood transfusions were necessary and all evaluations have been unremarkable. According to the guidelines of the American Academy of Pediatrics, at what age should you recommend initiation of iron supplementation for this infant? (check one) A. Now B. 1 month C. 3 months D. 6 months E. 9 months

According to the guidelines of the American Academy of Pediatrics, elemental iron supplementation (2 mg/kg per day) should begin at 1 month of age for exclusively breastfed infants born before 37 weeks gestation and should continue until 12 months of age, unless the infant had multiple blood transfusions.

A 30-year-old male is taking a motorcycle trip in 6 weeks to Colorado, including a ride to the top of Pikes Peak (elevation 14,100 ft). He has never been above 5000 ft prior to this trip and is concerned about developing acute mountain sickness (AMS). He is generally healthy and takes no medications, but smokes 1 pack of cigarettes per day. He is allergic to penicillin. Which one of the following is the best option for this patient to reduce his risk of developing AMS? (check one) A. An intensive aerobic fitness program B. Varenicline (Chantix) for smoking cessation C. Prophylaxis with acetazolamide D. Prophylaxis with Ginkgo biloba

Acute mountain sickness (AMS) occurs in at least 25% of persons traveling to destinations over 8000 feet above sea level. Risk factors include rapid ascent, living at low altitudes (<2000 ft), a prior history of altitude illness, and strenuous physical exertion during the ascent. AMS is most often manifested by headache, fatigue, lightheadedness, and/or nausea. The best way to prevent AMS is gradual ascent, but medications may also be effective in prophylaxis, especially if a rapid ascent such as in motorcycling, driving, or flying to altitude is planned. The drug of first choice in preventing AMS is acetazolamide, a carbonic anhydrase inhibitor, starting the day before ascent. It is, however, contraindicated in patients with sulfa allergy. The second-line drug for prevention is dexamethasone, which should be used for prophylaxis in sulfa-allergic patients. It is also used in the treatment of AMS and high-altitude cerebral or pulmonary edema, but immediate descent of at least 2000 feet is imperative if either of those more serious complications develop. While advocated as a prophylactic and treatment option for AMS, the results for ginkgo are mixed and it is therefore not recommended for use in this situation. Smoking cessation and physical conditioning are both good ideas for this patient, but neither will reduce his risk for developing AMS. Ref: Fiore DC, Hall S, Shoja P: Altitude illness: Risk factors, prevention, presentation, and treatment. Am Fam Physician 2010;82(9):1103-1110. 2) Nieto Estrada VH, Molano Franco D, Medina RD, et al: Interventions for preventing high altitude illness: Part 1. Commonly-used classes of drugs. Cochrane Database Syst Rev 2017;(6):CD009761.

An elderly female who has been your patient for several years is discovered lying on the floor of her kitchen by a Meals on Wheels volunteer. She is transported to the hospital in an unresponsive state. After a thorough evaluation you diagnose a massive cerebral infarct. On several previous occasions the patient verbalized to you her desire to not be subjected to life-prolonging treatments should she ever be rendered incapacitated but she declined your suggestion that she confirm this in writing. The patient is admitted to the hospital with "do not resuscitate" (DNR) orders and supportive measures are instituted. A neurology consultant evaluates her and agrees that her condition is terminal and irreversible. The patient's nephew, who is her closest relative, is angered by the DNR status and, noting that she has Medicare coverage, demands every medical treatment that might prolong his aunt's life, including resuscitation. Which one of the following would be the best course of action in terms of legality and ethics? (check one) A. Institution of aggressive medical therapies, including full resuscitation in the event of cardiac arrest and ventilator support in the event of respiratory arrest B. Continuation of a supportive treatment plan, provided full resuscitation is initiated in the event of cardiac arrest and ventilator support is provided in the event of respiratory arrest C. Performance of full CPR for cardiopulmonary arrest, but no ventilator support for respiratory failure D. Continuation of the current treatment plan

Adult patients or their legally authorized representatives have the right to refuse any medical treatment, regardless of its likelihood of success; however, there is no legal right to receive any and all treatment demanded. When patients have explicit advance directives in writing, their wishes are clear. When no written document exists but the patient's desire is well known to his or her physician, the physician is ethically bound to honor these wishes. This responsibility must be balanced against the physician's ethical obligation not to perform futile treatments of no benefit to the patient. Family members acting as a medical decision-making proxy are obligated to represent what they believe to be the patient's wishes, even if this conflicts with their personal beliefs. This case highlights two commonly encountered issues: honoring the patient's wishes when there is conflicting evidence of what their wishes may be, and withholding medically futile treatment. In this case, the futility of treating the patient's condition overrides any confusion as to her wishes, and the admission treatment plan is the most appropriate. Nevertheless, it would be best to discuss this with the nephew and attempt to arrive at a consensus, with the patient's best interest being the primary concern.

A nursing home resident is hospitalized, and shortly before she is to be discharged she develops a skin ulcer that proves to be infected with MRSA. Which one of the following is most important in terms of infection control when she returns to the nursing home? (check one) A. Surveillance cultures of residents with rooms near the patient B. Aggressive housekeeping in the patient's room C. Masks, gowns, and gloves for anyone entering the patient's room D. Strict handwashing practices by all staff, visitors, and residents E. Isolation of the patient in a room by herself

All staff, visitors, and nursing home residents should observe strict handwashing practices when a resident has a MRSA infection. Barrier precautions for wounds and medical devices should also be initiated. Surveillance cultures are not warranted. Aggressive housekeeping practices play little, if any, role in preventing the spread of MRSA. Isolating the patient is not practical or cost effective.

While making rounds at a nursing home you see a 70-year-old female with dementia. The staff tells you that she has recently developed serious aggressive behaviors that include lashing out physically at caregivers on a regular basis. Nonpharmacologic interventions have not curbed her violent outbursts. Your evaluation does not reveal any treatable underlying conditions. After a conversation about risks and benefits with her family and the nursing home staff, which one of the following would you recommend for this patient? (check one) A. Diphenhydramine (Benadryl) B. Aripiprazole (Abilify) C. Clonazepam (Klonopin) D. Mirtazapine (Remeron) E. Ziprasidone (Geodon)

Although the FDA has not approved the use of antipsychotics for aggressive behavior associated with dementia, they are often used to treat refractory behavioral and psychological symptoms of dementia. Their off-label use should be considered only when nonpharmacologic therapies are ineffective and the behaviors pose a risk of harm to the patient or others (SOR C), and the drug should be discontinued if there is no evidence of symptom improvement (SOR A). In a meta-analysis of three atypical antipsychotics, only aripiprazole showed small average reductions in behavioral and psychological symptoms of dementia. Olanzapine has demonstrated inconsistent results and ziprasidone is ineffective. Diphenhydramine is an anticholinergic agent and could exacerbate behaviors. Mirtazapine is indicated for depression. The American Geriatrics Society recommends against the use of benzodiazepines in older adults as a first choice for insomnia, agitation, or delirium. Ref: Kalish VB, Gillham JE, Unwin BK: Delirium in older persons: Evaluation and management. Am Fam Physician 2014;90(3):150-158. 2) Reese TR, Thiel DJ, Cocker KE: Behavioral disorders in dementia: Appropriate nondrug interventions and antipsychotic use. Am Fam Physician 2016;94(4):276-282.

A 45-year-old female comes to your office for follow-up of palpitations and anxiety. Her pulse rate is 112 beats/min and her heart rate is regular with no audible murmurs. Her thyroid is nontender with no palpable nodules. You note bilateral exophthalmos as well as pretibial myxedema. Her TSH level is 0.05 μU/mL (N 0.5-5.0); free T4 and total T3 levels are elevated. A pregnancy test is negative.Which one of the following would be the best option for limiting the progression of this patient's ophthalmopathy? (check one) A. Atenolol (Tenormin) B. Cholestyramine (Questran) C. Methimazole (Tapazole) D. Prednisone E. Radioactive iodine

Based on her clinical presentation and classic ophthalmopathy, this patient has Graves disease. Unlike radioactive iodine, methimazole has been shown to decrease the risk of development or progression of ophthalmopathy in Graves disease (SOR B). Atenolol is used for symptomatic control in hyperthyroidism. Cholestyramine can help lower thyroid hormone acutely but is not a long-term treatment. Prednisone is used for severe hyperthyroidism and not long-term treatment. Atenolol, cholestyramine, and prednisone do not have any effect on the long-term complications of Graves disease.

An 11-year-old female is brought to your office for a routine well child examination. The American Academy of Pediatrics recommends screening this patient for which one of the following? (check one) A. Anemia B. Diabetes mellitus C. Dyslipidemia D. HIV

Because of concerns about the growing epidemic of obesity in this population, the American Academy of Pediatrics now recommends screening for elevated serum cholesterol levels in children 9-11 years of age (SOR C). This patient should also be screened annually for depression beginning at 11 years of age and continuing through 21 years of age. Universal screening for iron deficiency anemia is recommended at 12 months of age and again at 15-30 months of age if the patient is determined to be high risk. HIV screening is recommended in adolescents 16-18 years of age, and age 21 is now the recommended starting age for screening for cervical dysplasia. Universal screening for diabetes mellitus is not recommended for children or adolescents.

A 13-year-old female sees you for the first time. Her mother has brought her in because the patient has never menstruated. She feels well and has no other health concerns. Her medical history is unremarkable, and she takes no medications. A physical examination is significant for the absence of both breast development and pubic hair. Her height is below the 2nd percentile and her weight is at the 6th percentile for her age. β-hCG, prolactin, and TSH levels are all normal. LH and FSH levels are elevated. Which one of the following would you order at this point? (check one) A. A trial of combined oral contraceptive pills B. A serum testosterone level C. Pelvic ultrasonography D. Karyotyping E. MRI of the brain and brainstem

Brain MRI would be useful in ruling out brain and pituitary tumors, but this patient has no brain tumor symptoms and normal prolactin and TSH levels. A trial of combined oral contraceptive pills may precipitate withdrawal bleeding, but this has poor sensitivity as a marker of ovarian function (SOR C) and will not lead to a diagnosis. There are no signs or symptoms of hyperandrogenism, so a serum testosterone level would not be helpful. Pelvic ultrasonography to confirm the presence of a uterus is not indicated by the physical findings at this point. Short stature and the lack of pubertal changes are characteristics of primary ovarian failure and Turner syndrome. A karyotype to rule out Turner syndrome and the presence of Y chromosome material is indicated at this point.

A 30-year-old female presents to your office for evaluation of a 5-mm pulmonary nodule noted on CT of the chest performed after a motor vehicle accident 2 weeks ago. She has had no symptoms and she is not a smoker. A physical examination is unremarkable.Which one of the following would be most appropriate at this point? (check one) A. Noncontrast chest CT in 1 year B. A PET scan C. Referral for a needle biopsy D. Referral for bronchoscopy E. Referral for wedge resection

By definition, a pulmonary nodule is a circumscribed, round lesion that may measure up to 3 cm in size and is surrounded by aerated lung. Management is based on the size of the nodule and the probability of malignancy. Risk factors for lung cancer include a previous malignancy, a positive smoking history, and age ≥65. Only 1% of nodules between 2 mm and 5 mm in size are malignant. Nodules <8 mm are difficult to biopsy, and a PET scan is not reliable. The risk of surgery outweighs the benefits in nodules of this size. For a low-risk patient with a nodule 4 mm to <6 mm in size, a repeat noncontrast CT at 12 months is recommended. If it is unchanged, no further follow-up is needed.

A mother brings in her 11-year-old daughter, stating that the child has not had a bowel movement in 5 days. Although she is very embarrassed to talk to you, the daughter confirms that this is the case, and that it has happened several times since she started middle school earlier this year, where the bathrooms are very unpleasant. Both the mother and the daughter state that this has not been a problem in the past. The mother reports that the daughter is otherwise healthy, takes no medications, and has no past surgical history. The patient has no discomfort with urination but does have some abdominal pain that has grown worse over the last day or so. She is not having any trouble breathing. She reports a decreased appetite over the last 3 days. The physical examination reveals normal vital signs, a normal BMI, and no abdominal distention or tenderness to palpation. Which one of the following would you advise at this point? (check one) A. Observation only, as this problem will resolve on its own B. Dedicated "toilet time" before and after school and nightly before bed C. A phosphate soda (Fleet) enema nightly D. Polyethylene glycol (MiraLAX) E. An abdominal radiograph

Childhood constipation is a very common problem encountered in outpatient practice. Most childhood constipation is functional, which appears to be the case with this patient. She has no evidence of organic disease, and she had a change in lifestyle that caused her to retain stool, resulting in a cycle of constipation. Polyethylene glycol is easily tolerated, safe, and easy to use. For this reason, it is the first-line therapy. A daily dose should resolve this issue, although it is likely that the child will need another course of treatment, as functional constipation is likely to recur. It is not reasonable in this case to provide no treatment, as the child is uncomfortable. An enema would be unnecessary since oral therapies are very likely to be effective. Behavioral therapies have not been shown to be as effective as laxatives for functional constipation, and stringent therapy that is not related to food intake is likely to increase the stress surrounding going to the toilet. There is no role for imaging in a case where the suspicion of an organic cause is so low.

A 12-year-old male has a 1-week history of fever, headache, sore throat, and a mildly productive cough. He also began having ear pain yesterday. On examination he does not appear to be toxic. He has a temperature of 37.8°C (100.0°F). Examination of his ears shows a bulla on the right tympanic membrane, as well as mild to moderate erythema of the posterior pharynx. The neck is supple. The lungs have a few scattered crackles. The remainder of the examination is unremarkable. A chest radiograph reveals thickened bronchial shadows, as well as interstitial infiltrates in the lower lobes.The most appropriate treatment at this time would be (check one) A. amoxicillin B. azithromycin (Zithromax) C. ceftriaxone (Rocephin) D. cefuroxime (Zinacef) E. vancomycin

Community-acquired pneumonia in children is treated based on age. The most likely etiologic agents in a school-age child are Mycoplasma pneumoniae , Chlamydia pneumoniae , and Streptococcus pneumoniae . Group A Streptococcus and Haemophilus influenzae are less common causes. Staphylococcus aureus that is methicillin-resistant has become increasingly common. The preferred treatment for community-acquired pneumonia is a macrolide antibiotic such as azithromycin. In children ages 5-16, Mycoplasma pneumonia tends to have a gradual onset of symptoms and seldom causes respiratory distress. Signs and symptoms may vary. The patient may develop a rash, musculoskeletal symptoms, or gastrointestinal symptoms. Radiographs may reveal bronchopneumonia, nodular infiltrates, hilar adenopathy, pleural effusions, or plate-like atelectasis. Ear pain may be due to bullous myringitis, although this may be viral as well. Laboratory findings may not be helpful, as the WBC count may be normal or slightly elevated. There may be thrombocytosis, an elevated erythrocyte sedimentation rate, an elevation of cold agglutinins, or an elevated reticulocyte count. A Coombs test is seldom needed, although it might be helpful at times. The diagnosis is generally made on a clinical basis.

A 54-year-old male plumber presents with a 2-month history of nonpainful swelling in the elbow. He has no history of injury or trauma. An examination is significant for a nontender, soft nodule over the olecranon process with no erythema or warmth. Which one of the following would be most appropriate at this point? (check one) A. Conservative management with ice, compression wraps, and activity modification B. Broad-spectrum antibiotics with coverage for MRSA C. An intrabursal corticosteroid injection D. An intrabursal hyaluronic acid (Synvisc) injection E. Aspiration of the bursa fluid under sterile conditions

Conservative treatment is the recommended initial management for olecranon bursitis when there is no history of trauma or signs of septic bursitis. Aspiration of the bursal fluid is not recommended initially due to the risk of iatrogenic infection, but can be considered for symptomatic relief if there is significant enlargement or symptoms, or for diagnosis and culture if septic bursitis is suspected. Antibiotics are not recommended for aseptic bursitis and should be delayed in septic bursitis until after aspiration for culture. MRSA coverage may be indicated if the patient is at high risk for MRSA infection. An intrabursal corticosteroid injection is not routinely recommended for bursitis unless an underlying inflammatory condition is suspected, such as gout or rheumatoid arthritis. An intrabursal hyaluronic acid injection is not a recommended treatment for bursitis.

An adolescent male is concerned because he is the shortest boy in his class. His age is 14.3 years and his parents are of normal height. He has a negative past medical history and no symptoms. On examination he is 151 cm (59 in) tall. The average height for his age is 165 cm. His weight is 43 kg (95 lb). His sexual maturity rating is 3 for genitalia and 2 for pubic hair. A wrist radiograph shows a bone age of 12.2 years. The average height for this bone age is 152 cm.On the basis of this evaluation you can tell the patient and his parents that (check one) A. he should have a growth hormone stimulation test B. his adult height will be below average C. his sexual development is about average for his age D. he will begin to grow taller within approximately a year E. an underlying nutritional deficiency may be the cause of his short stature

Constitutional growth delay, defined as delayed but eventually normal growth in an adolescent, is usually genetic. If evaluation of the short adolescent male reveals no evidence of chronic disease, if his sexual maturity rating is 2 or 3, and if his height is appropriate for his skeletal age he can be told without endocrinologic testing that he will begin to grow taller within a year or so. His adult height may be below average but cannot be predicted reliably. Average sexual maturity ratings for a male of 14.3 years are 4 for genitalia and 3-4 for pubic hair. The history and physical examination would have given clues to any illnesses or nutritional problems.

A 10-year-old female is brought to your office by her parents on the recommendation of the school counselor. Her parents report that for at least the past year, both at home and at school, their daughter often loses her temper, is easily annoyed, and is "very touchy." She is sullen and angry, arguing frequently with her parents and teachers. At school she irritates and annoys classmates, then blames them when they react negatively toward her. She also disrupts the classroom by refusing to comply with classroom rules and expectations or with the teacher's authority. This child's behavior is most consistent with which one of the following? (check one) A. Attention-deficit/hyperactivity disorder B. Bipolar disorder C. Conduct disorder D. Intermittent explosive disorder E. Oppositional defiant disorder

Correct This child presents with oppositional defiant disorder (ODD). To meet the DSM-5 criteria for ODD, the child must demonstrate at least four symptoms from any of the following categories: angry/irritable mood (often loses temper, is often touchy or easily annoyed, is often angry and resentful), argumentative/defiant behavior (often argues with authority figures or with adults, often actively defies or refuses to comply with requests from authority figures, often deliberately annoys others, often blames others for his/her mistakes or misbehavior), and vindictiveness (has been spiteful or vindictive at least twice within the past 6 months). These behaviors must be directed toward at least one person other than a sibling. Behavioral problems associated with conduct disorder are more severe, including aggression toward animals or other persons, destruction of property, and a pattern of theft or deceit. The person's conduct frequently leads to conflict with authority figures. Attention-deficit/hyperactivity disorder involves difficulty in following rules, struggles with authority figures, and possibly annoying others. In ODD, however, defiance of authority figures occurs in settings other than those where sustained attention or sitting quietly is required. Bipolar disorder can include irritability and negative affect but the argumentative, defiant behavior or vindictiveness that occur in ODD do not routinely occur in mood disorders. Intermittent explosive disorder involves repeated, sudden episodes of impulsive, aggressive, violent behavior or angry verbal outbursts in which the person reacts grossly out of proportion to the situation. Road rage, domestic abuse, throwing or breaking objects, or other temper tantrums may be signs of intermittent explosive disorder. Serious aggression toward others does not occur in ODD.

A 33-year-old female comes to your office for follow-up of irritable bowel syndrome. You ruled out other causes of her abdominal bloating, abdominal pain, and diarrhea at earlier visits. She has no change in symptoms, such as constipation or blood in her stool. She has resisted treatment in the past, but her symptoms are becoming more frequent and she would now like to consider treatment. Evidence shows that which one of the following would most likely be beneficial for this patient? (check one) A. Acupuncture B. Increased insoluble fiber in her diet C. Fluoxetine (Prozac), 20 mg daily D. Neomycin, 1000 mg every 6 hours for 7 days E. Polyethylene glycol (MiraLAX), 17 g daily

Correct This patient has diarrhea-predominant irritable bowel syndrome (IBS). There are many treatments available, with varying degrees of evidence. SSRIs, along with tricyclic antidepressants, have been shown to decrease abdominal pain and improve global assessment scores in those with IBS. Polyethylene glycol is a treatment for constipation and would not help this patient. Acupuncture has not been shown to be superior to sham acupuncture in improving IBS symptoms. Neomycin has been shown to improve symptoms in constipation-predominant IBS but would not be helpful in diarrhea-predominant IBS. Soluble fiber such as psyllium improves symptoms and decreases abdominal pain scores in patients with IBS. Insoluble fiber has not been shown to improve any IBS outcomes.

A 75-year-old male with a history of hypertension, TIA, and atrial fibrillation sees you for follow-up. Ten days ago he was on vacation in another state when he developed chest pain. He went to a local hospital where he was diagnosed with an ST-elevation myocardial infarction (STEMI) and was taken immediately for cardiac catheterization. He had a drug-eluting stent placed in his left anterior descending artery. He brings some discharge paperwork with him, including a medication list, but has not yet seen a local cardiologist. He is concerned that he is taking too many blood thinners. He feels well and does not have any chest pain, shortness of breath, or excessive bleeding or bruising.Prior to his STEMI the patient was taking lisinopril (Prinivil, Zestril), 10 mg daily; warfarin (Coumadin), 2.5 mg daily; and metoprolol succinate (Toprol-XL), 25 mg daily. Upon discharge he was instructed to continue all of those medications and to add clopidogrel (Plavix), 75 mg daily, and aspirin, 81 mg daily.The patient's vital signs and physical examination are normal except for an irregularly irregular rhythm on the cardiovascular examination. His INR is 2.5.Which one of the following would be most appropriate at this time? (check one) A. Continue the current regimen B. Discontinue aspirin C. Discontinue clopidogrel D. Discontinue warfarin E. Decrease warfarin with a goal INR of 1.5-2.0

Current guidelines recommend that patients with an ST-elevation myocardial infarction (STEMI) who also have atrial fibrillation take dual antiplatelet therapy such as aspirin plus clopidogrel and a vitamin K antagonist, with a goal INR of 2.0-3.0. If a patient was already taking a direct-acting oral anticoagulant (DOAC) instead of warfarin for atrial fibrillation, the patient should continue with the DOAC in addition to dual antiplatelet therapy. The duration of triple therapy should be as short as possible, and aspirin can often be discontinued after 1-3 months. However, this patient's STEMI occurred less than 2 weeks ago and he should continue triple therapy.

A 26-year-old male graduate student presents with concerns about having attention-deficit disorder (ADD). He reports inattentiveness and a lack of concentration for the last 2-3 years, resulting in poorer academic performance than when he was an undergraduate. He says his wife also complains of his lack of focus and attentiveness when attending to household responsibilities. He does not recall having similar symptoms in grade school or high school.You explain that in order to diagnose adult ADD (check one) A. evidence of symptoms of ADD must be present before age 6 B. evidence of symptoms of ADD must be present before age 12 C. a therapeutic trial of a short-acting dextroamphetamine/amphetamine combination (Adderall) is indicated D. a therapeutic trial of an SNRI such as atomoxetine (Strattera) is indicated

Diagnosing attention-deficit disorder in adults requires symptoms that interfere with social, academic, or occupational functioning and are present in more than one setting. DSM-5 states that a history of symptoms before age 12 is required for the diagnosis. DSM-IV specified that symptoms must have been present before age 7.

A 72-year-old female taking hydrochlorothiazide for hypertension develops trigeminal neuralgia and you start her on carbamazepine (Tegretol). She is at risk for which one of the following metabolic consequences? (check one) A. Calcium pyrophosphate deposition B. Hypercalcemia C. Hyponatremia D. Hyperuricemia

Elderly patients, especially those taking hydrochlorothiazide, are at risk for developing hyponatremia while taking carbamazepine. Carbamazepine is one of the medications that can cause the syndrome of inappropriate antidiuretic hormone secretion, as it interferes with the ability to dilute the urine. It does not lead to the other derangements listed (SOR A).

You order an NT-proBNP level in a patient with symptoms and signs of heart failure. Which one of the following would contribute to a result that is higher than expected? (check one) A. Male sex B. Elevated BMI C. Elevated albumin D. Elevated creatinine

Elevated levels of NT-proBNP are known to indicate an increased likelihood of heart failure, and lower levels can rule out heart failure. However, certain patient characteristics can lead to higher levels of NT-proBNP even in healthy individuals. The use of one normal cutoff level for elevated NT-proBNP may not be appropriate. Even healthy female patients and those >65 years of age will have higher levels of NT-proBNP than younger male patients (SOR A).NT-proBNP is negatively correlated with kidney function as measured by the estimated glomerular filtration rate (GFR) and albumin levels. Patients with a low GFR or a low level of albumin have higher NT-proBNP levels (SOR A). Interestingly, grip strength is negatively correlated with NT-proBNP as well.A higher BMI is associated with a lower NT-proBNP. Thus, the utility of NT-proBNP to rule out heart failure in obese patients is decreased (SOR A).

A 50-year-old female comes to your office for routine health maintenance. She jogs 2 miles a day and has had left medial knee pain for the last 6 months. Radiographs reveal moderate degenerative arthritis of the knee. Her BMI is 24.1 kg/m2 and her physical examination, including an examination of the knee, is otherwise normal. Which one of the following would be most effective for this patient's arthritis? (check one) A. Weight loss B. A knee brace C. Foot orthoses D. Hyaluronic acid injection E. Exercise-based physical therapy

Exercise-based therapy is the foundation for treating knee osteoarthritis. Foot orthoses can be helpful for anterior knee pain but this patient's pain is located medially. The benefit of hyaluronic acid injections is controversial, and recommendations vary; recent systematic reviews do not support a clinically significant benefit. Weight loss is recommended for patients with a BMI >25.0 kg/m2. Wearing a knee brace has shown little or no benefit for reducing pain or improving knee function.

A 34-year-old sexually active female consults you about contraception options. She has late-stage kidney disease and her nephrologist has notified you that she will likely be recommended for kidney transplantation soon.Which one of the following would you recommend for safety and efficacy? (check one) A. Condoms B. Combined oral contraceptive pills C. Medroxyprogesterone acetate (Depo-Provera) injections D. An IUD E. An etonogestrel/ethinyl estradiol vaginal ring (NuvaRing)

Family physicians are often asked to provide primary care for organ transplant recipients. Pregnancy should be avoided during the 12 months following transplantation because of the increased risk of preterm delivery and graft rejection. Female fertility typically increases post transplant. The use of an IUD avoids interactions with medications, does not increase the risk of infection, and is not affected by typical immunosuppressive therapies. The remaining options are incorrect because of their higher failure and discontinuation rates. The CDC cites failure rates with typical use of 9% for combined oral contraceptives and the etonogestrel/ethinyl estradiol vaginal ring, 6% for injectable progesterone, 0.2% for levonorgestrel IUDs, and 0.08% for the copper IUD. Barrier method failure rates exceed 18%. Combined oral contraceptives and the vaginal ring also have potential estrogen-related side effects, and injectable progesterone use increases the risk for osteoporosis.

A 21-year-old female asks you about Papanicolaou (Pap) testing recommendations. You determine she is at average risk for cervical cancer and recommend which one of the following? (check one) A. Pap testing without HPV co-testing now and in 1 year if results are normal B. Pap testing without HPV co-testing now and in 3 years if results are normal C. Pap testing with HPV co-testing now and in 3 years if results are normal D. Pap testing with HPV co-testing now and in 5 years if results are normal E. HPV testing only, now and in 5 years if results are normal

Family physicians should be familiar with age-appropriate cancer screening recommendations. Deaths from cervical cancer have been significantly reduced through screening. HPV testing is not recommended for screening in average-risk women younger than 30 years old. Cytology without HPV testing is recommended for screening every 3 years for an average-risk 21-year-old female.

A 45-year-old female visits your office for her annual health maintenance visit and mentions that her hair has been thinning over the past few years. She is now concerned that it may be noticeable. She takes no medications and is otherwise healthy with normal menstrual cycles. On examination she has a negative pull-away test. You note diffuse thinning in the parietal regions, with sparing of the frontal hairline. She has no scalp scarring, scale, or erythema. Which one of the following would be the most appropriate pharmacotherapy? (check one) A. Finasteride (Propecia), 1 mg daily B. Griseofulvin, 500 mg daily C. Hydroxychloroquine (Plaquenil), 200 mg twice daily D. Minoxidil 2% (Rogaine), applied to the scalp twice daily E. Triamcinolone 0.05% (Trianex), applied to the scalp twice daily

Female pattern hair loss is categorized as diffuse and nonscarring. It presents with parietal hair thinning with preservation of the frontal hairline. Minoxidil 2% produces regrowth of hair in female pattern hair loss (SOR B). Oral finasteride is appropriate only for men with male pattern hair loss (SOR A). Hydroxychloroquine is used for inflammatory hair loss associated with discoid lupus erythematosus, which is focal and scarring. Topical corticosteroids are appropriate for alopecia areata (SOR B) but not for female pattern hair loss. Griseofulvin is used to treat tinea capitis, which presents as focal scale with erythema.

A 51-year-old female comes to your office for follow-up of fibromyalgia. She is currently taking amitriptyline, 10 mg at bedtime, and naproxen (Naprosyn), 500 mg twice daily, for her symptoms. A member of her fibromyalgia support group recommended fluoxetine (Prozac) to her and she asks you if it would be helpful.It would be appropriate to tell her that SSRIs for the treatment of fibromyalgia (check one) A. do not affect depression scores B. reduce fatigue C. provide some pain reduction D. help with sleep problems E. are superior to tricyclics for pain control

Fibromyalgia is a chronic complex condition characterized by muscle pain, fatigue, muscle tenderness, and sleep disorders, often accompanied by mood disorders. SSRIs have been studied in the treatment of these symptoms, and while they have been shown to produce up to a 30% reduction in pain scores in patients with fibromyalgia, they have not been shown to affect fatigue or sleeping problems. They also have not been shown to be superior to tricyclics when treating pain. As with other patient populations, SSRIs have been shown to improve depression in those with fibromyalgia.

A 53-year-old male sees you for follow-up of his hypertension. His medical history includes prediabetes and gout, and he is currently taking lisinopril (Prinivil, Zestril), 40 mg daily, to control his blood pressure. His blood pressure after resting is 148/86 mm Hg. Laboratory findings include a serum creatinine level of 0.8 mg/dL (N 0.6-1.2) and a serum potassiumlevel of 4.5 mEq/L (N 3.5-5.1).Which one of the following would be the most appropriate management of this patient's hypertension? (check one) A. No change in medication B. Add amlodipine (Norvasc) C. Add hydrochlorothiazide D. Add losartan (Cozaar) E. Add metoprolol succinate (Toprol-XL)

First-line agents for hypertension include ACE inhibitors, angiotensin receptor blockers, thiazide diuretics, and calcium channel blockers. Hydrochlorothiazide would be relatively contraindicated due to the patient's gout. Losartan, an angiotensin receptor blocker, should not be added because the patient is taking an ACE inhibitor. Metoprolol succinate, a β-blocker, is not a first-line agent for blood pressure unless there is another indication such as systolic heart failure or migraine prophylaxis.

A 16-year-old male is hit on his shoulder while playing football. A radiograph subsequently reveals a midclavicular fracture that is minimally displaced.Appropriate management would be (check one) A. a weekly radiograph to monitor for displacement B. a sling for 2-6 weeks C. a shoulder immobilizer for 6 weeks D. a figure-of-eight bandage for 8 weeks E. immediate referral to an orthopedist

Fractures of the clavicle are common in young individuals, usually from sports injuries or direct trauma. Eighty percent of these fractures occur in the midclavicle. Unless significantly displaced, these fractures do not require referral. They can be treated with just a sling for 2-6 weeks. A sling is more comfortable and less irritating than a figure-of-eight bandage. Passive range of motion of the shoulder is indicated as soon as the pain allows. Physical therapy may be started at 4 weeks after the injury.

GLP-1 agonists such as exenatide (Byetta) can be used as second-line agents to help improve glycemic control in patients with type 2 diabetes mellitus. Which one of the following is a CONTRAINDICATION to their use? (check one) A. Hypothyroidism B. Thyroid cancer C. Coronary artery disease D. Heart failure

GLP-1 agonists are contraindicated in patients with medullary thyroid cancer or multiple endocrine neoplasm syndrome, or with a family history of these conditions. They are not associated with heart failure, coronary artery disease, or hypothyroidism. They have been associated with pancreatitis in rare cases, but this is not a contraindication to prescribing them. Ref: Two new GLP-1 receptor agonists for diabetes. Med Lett Drugs Ther 2014;56(1455):109-111.

A 7-month-old infant is hospitalized for the third time with lower-lobe bronchopneumonia. Findings include a weight and height below the 10th percentile. A sibling died of sudden infant death syndrome.Laboratory testing reveals a hemoglobin level of 9.0 g/dL (N 10.5-14.0), a mean corpuscular volume of 85 μm3 (N 72-88), and a serum calcium level of 9.0 mg/dL (N 9.0-10.5). A sweat chloride level is 20 mEq/L (N <60).Which one of the following is the most likely cause of this infant's failure to thrive? (check one) A. Cystic fibrosis B. DiGeorge's syndrome C. Battered child syndrome D. Gastroesophageal reflux E. β-Thalassemia

Gastroesophageal reflux accounts for a significant number of cases of failure to thrive, crib death, and recurrent pneumonia. Features of gastroesophageal reflux include a history of recurrent pneumonia, a low growth curve, a family history of sudden infant death syndrome, and normocytic anemia. A sweat chloride level of 20 mEq/L rules out cystic fibrosis. Normal serum calcium excludes DiGeorge's syndrome. The battered child generally presents with more than just a single recurring medical problem. β-Thalassemia would be indicated by a microcytic anemia.

A 74-year-old female comes to the emergency department with the acute onset of severe pain in the middle to lower back after lifting a small piece of furniture. Imaging demonstrates an acute nondisplaced wedge compression fracture of the T12 vertebra. An examination confirms that there is no neurologic deficit. She is admitted to the hospital.In addition to pain management, which one of the following would you recommend? (check one) A. Early mobilization as tolerated B. Bed rest until a back brace is obtained C. Evaluation for kyphoplasty D. Evaluation for vertebroplasty E. A neurosurgical evaluation

Generally, the goal for patients with vertebral compression fractures is early mobilization when tolerated. Bed rest is ordered only if movement is not tolerated. The evidence for back bracing is limited but it can be used after weighing the risks and benefits. Current evidence supports initial conservative treatment before considering vertebroplasty or kyphoplasty (SOR C). Neurosurgical consultation is not required in this case.

One of your patients recently went into atrial fibrillation and you order an echocardiogram. The diameter of which one of the following structures best predicts the likelihood that sinus rhythm will be maintained after successful cardioversion? (check one) A. The left atrium B. The right atrium C. The left ventricle D. The right ventricle E. The aortic root

If atrial fibrillation is converted back to sinus rhythm, the likelihood of the patient staying in sinus rhythm is best predicted from the diameter of the left atrium on the patient's echocardiogram. Significant left atrium enlargement means the patient is unlikely to stay in sinus rhythm after successful conversion. Other factors that predict a lack of success in maintaining sinus rhythm after cardioversion include a longer time in atrial fibrillation before cardioversion, or the presence of underlying heart disease, especially rheumatic heart disease.

You see a 12-year-old female for a well child check. She is healthy without any medical problems and neither she nor her father have any concerns today. She is up to date on her immunizations except for her third dose of HPV vaccine. She received the first dose of the vaccine at her 11-year-old well child check and her second dose 1 month later. However, she was told by a few friends that they only needed to get two doses of HPV vaccine, so she is very excited that she does not need any shots today. According to the CDC, which one of the following is true regarding HPV vaccine for this patient? (check one) A. She has completed her HPV vaccine series B. She should receive a third dose of HPV vaccine today C. She needs an HPV booster at age 21 but does not need a third dose of vaccine today D. She should have HPV titers drawn today and receive a third dose of vaccine only if the titers are low

In 2016 the CDC changed the recommendation for the number of HPV vaccine doses for children ages 11-14. Children in this age group need only two doses of HPV vaccine 6-12 months apart. However, if they received two doses of HPV vaccine less than 5 months apart, they still need to have the third dose. Children and young adults over the age of 14 and those with certain immunocompromising conditions still require three doses of HPV vaccine. There is no indication for a booster dose at a later date, nor is there clinical data to support using titers to gauge immunogenicity to HPV.

A 63-year-old male hospitalized for treatment of metastatic lung cancer develops a proximal deep vein thrombosis of the leg. Which one of the following anticoagulants should be prescribed when he is discharged from the hospital? (check one) A. Apixaban (Eliquis) B. Dabigatran (Pradaxa) C. Enoxaparin (Lovenox) D. Rivaroxaban (Xarelto) E. Warfarin (Coumadin)

In a patient with cancer, deep vein thrombosis of the leg or a pulmonary embolus is considered to be cancer-associated thrombosis. Low molecular weight heparin (LMWH), such as enoxaparin, should be chosen over the other anticoagulant options listed. (LMWH over warfarin is a Grade 2B recommendation; LMWH over dabigatran is a Grade 2C recommendation; LMWH over rivaroxaban is a Grade 2C recommendation; and LMWH over apixaban or edoxaban is a Grade 2C recommendation). Ref: Kearon C, Akl EA, Ornelas J, et al: Antithrombotic therapy for VTE disease: CHEST guideline and expert panel report. Chest 2016;149(2):315-352.

A 75-year-old male presents with a 12-month history of chest pressure radiating to his left arm that occurs predictably after he walks briskly for 2 blocks and goes away with rest. A treadmill stress test suggests coronary artery disease. The patient would prefer medical therapy over revascularization if possible. The patient's blood pressure is 120/85 mm Hg. His heart rate is 52 beats/min and has been in the low 50s at past visits. You initiate daily aspirin and a high-intensity statin, and prescribe sublingual nitroglycerin to use as needed for chest pain. Which one of the following additional treatments is recommended for management of his angina? (check one) A. Isosorbide mononitrate B. Ivabradine (Corlanor) C. Metoprolol succinate (Toprol-XL) D. Ranolazine (Ranexa) E. Verapamil (Calan)

In addition to aspirin, a high-intensity statin, and sublingual nitroglycerin as needed, patients with chronic stable angina may be treated with β-blockers, calcium channel blockers, and/or long-acting nitrates. β-Blockers and heart rate-lowering calcium channel blockers should be avoided in this patient who already has bradycardia. Ranolazine, which affects myocardial metabolism, is not used as a first-line agent. Ivabradine is not a first-line agent and is used only in patients with heart failure. A long-acting nitrate or a dihydropyridine calcium channel blocker would be appropriate for this patient.

A 78-year-old female with Alzheimer's disease is accompanied to an office visit by her daughter. The daughter has asked to complete an advance directive giving her medical power of attorney. Which one of the following would indicate that the patient lacks capacity to make decisions with regard to completing her medical directive? (check one) A. A dementing illness B. Inconsistent answers to questions C. Lack of orientation to time D. Asking that her son make medical decisions for her instead of her daughter E. A score of 24/30 on the Mini-Mental State Examination

In order for patients to show they have the capacity to make a decision they must demonstrate an understanding of the situation, including the risks, benefits, and consequences of the decision or refusal of care. If a patient gives inconsistent answers to questions after multiple explanations, this indicates that there is a lack of understanding and would meet one of the criteria to determine that the patient lacks the capacity to make that decision. The presence of dementia can be associated with an increased incidence of having a lack of capacity; however, a diagnosis of dementia by itself does not indicate that the patient lacks the capacity to make a decision. While disorientation to time or a lower score on the Mini-MentalState Examination is associated with an increased risk of lacking capacity, these findings alone would not be enough to determine that the patient lacks capacity. The patient asking that her son be her medical decision maker instead of her daughter would not be an indication that she lacks capacity.

A 64-year-old male presents to your office after vomiting blood twice over the last 2 hours. He is healthy otherwise, except for hyperlipidemia. On the initial examination he has mild hypotension and tachycardia, which normalize after fluid resuscitation. He has no further vomiting episodes.What level of hemoglobin should be the threshold for transfusion of red cells in this situation? (check one) A. 6.0 g/dL B. 7.0 g/dL C. 8.0 g/dL D. 9.0 g/dL E. 10.0 g/dL

In otherwise healthy stable patients with upper gastrointestinal bleeding, a transfusion of red cells is recommended when the hemoglobin level falls below 7.0 g/dL. In hypotensive patients with severe bleeding, a blood transfusion before the hemoglobin level reaches 7.0 g/dL is needed to prevent significant decreases below this level that would occur with just fluid resuscitation. In hemodynamically stable patients with known cardiovascular disease and significant upper gastrointestinal bleeding, 8.0 g/dL should be the threshold for a blood transfusion.

A 30-year-old male presents to your office with a 10-day history of rhinorrhea, nasal congestion, cough, and headache. He initially had a low-grade fever that resolved, but over the past 2 days all of his symptoms have gotten worse. His past medical history is unremarkable and he does not smoke. On examination there is a purulent secretion noted from the right nasal cavity and tenderness over the right maxillary sinus region. The most appropriate treatment is (check one) A. amoxicillin/clavulanate (Augmentin) B. azithromycin (Zithromax) C. cefuroxime (Ceftin) D. levofloxacin (Levaquin) E. trimethoprim/sulfamethoxazole (Bactrim)

In the first 3 to 4 days of illness, viral rhinosinusitis cannot be distinguished from early acute bacterial rhinosinusitis. If the patient seems to be improving and then symptoms start to worsen on days 5-10 of the illness (double sickening), acute bacterial rhinosinusitis should be suspected. The color of the nasal discharge should not be used as the sole indication for antibiotic therapy. One study showed that unilateral predominance with purulent rhinorrhea had an overall reliability of 85% for diagnosing sinusitis. After 10 days of upper respiratory symptoms, the probability of acute bacterial rhinosinusitis is 60%. Antibiotic therapy should be considered if the patient does not improve after 7-10 days from the onset of symptoms or if the symptoms worsen at any time. According to most guidelines, the first-line antibiotic for treatment of adults with sinusitis is amoxicillin/clavulanate. Respiratory fluoroquinolones are not recommended as first-line medications, as they offer no additional benefits and have significant side effects. Second and third-generation cephalosporins, trimethoprim/sulfamethoxazole, and macrolide antibiotics are no longer recommended for initial therapy. This is due to high rates of resistance in Streptococcus pneumoniae and Haemophilus influenzae .

Which one of the following is true regarding direct observational therapy (DOT) in the treatment of active tuberculosis? (check one) A. It guarantees patient compliance with the prescribed regimen B. It is recommended only in the office or clinic setting C. It decreases drug-resistant tuberculosis D. Patients require less monitoring for signs of treatment failure

In the treatment of active tuberculosis, direct observational therapy (DOT) involves providing the antituberculosis drugs directly to patients and watching them swallow the medication. It is the preferred care management strategy for all patients with tuberculosis. The use of DOT does not guarantee the ingestion of all doses of every medication, as patients may miss appointments, may not actually swallow the pills, or may regurgitate the medication, sometimes deliberately. Due to these limitations, the use of DOT does not remove the need to monitor patients for signs of treatment failure. DOT is effective in a wide variety of settings, including in the community with health nurses. It even shows benefit when the observation makes use of telehealth settings or mobile phones. Among the important benefits of DOT are that it has been shown to decrease both the acquisition and transmission of drug-resistant tuberculosis and to increase treatment success in HIV-positive patients.

A 36-year-old female has had elevated blood pressure readings since establishing care with you 6 months ago. You have increased her antihypertensive therapy monthly in an attempt to treat her hypertension, and she is currently taking the maximum dosage of three antihypertensive medications from different classes. She confirms that her blood pressure is also elevated at home, typically ranging from 155/92 mm Hg to 165/98 mm Hg. She is otherwise well. She does not have chest pain, shortness of breath, headaches, daytime sleepiness, or lower extremity edema. Her family history is significant for hypertension diagnosed in her father in his 50s.On examination the patient has a blood pressure of 168/95 mm Hg. Her pulse rate is 78 beats/min and her BMI is 28.1 kg/m2. She has a normal cardiac examination and no peripheral edema. You order laboratory testing, with the following significant findings: Sodium................................................144 mEq/L (N 136-142)Potassium...........................................3.0 mEq/L (N 3.5-5.0)Creatinine............................................0.72 mg/dL (N 0.6-1.2)Fasting glucose................................ 98 mg/dL TSH..................................................... 1.46 μU/mL (N 0.4-4.2) EKG..................................................... normal Which one of the following additional tests is most likely to reveal the cause of her hypertension? (check one) A. A dexamethasone suppression test B. Plasma aldosterone/renin activity C. Plasma free metanephrines D. A sleep study E. Renal ultrasonography

In young patients with hypertension it is important to consider secondary causes in addition to the more common essential hypertension. This patient's relatively young age and elevated home blood pressure readings despite drug therapy warrant further evaluation. The initial evaluation showed hypokalemia, which suggests an endocrine cause of hypertension, specifically hyperaldosteronism. Other potential causes of secondary hypertension include coarctation of the aorta, renal artery stenosis, thyroid disorders, obstructive sleep apnea, pheochromocytoma, and Cushing syndrome. Each of these presents with clinical findings that help to distinguish them from other potential causes, and the laboratory evaluation would depend on the suspected cause.

A mother who recently immigrated to the United States from Mexico brings her 4½-year-old son to your clinic for his pre-kindergarten examination. The child's examination is normal except for a hemangioma located on his left arm. His mother says that the lesion appeared at about 4 weeks of age, continued to grow until he was about 5 months of age, and then began to flatten, shrink, and fade. She is concerned because it has not improved in the past 18 months. When you examine the lesion more closely you note telangiectasia, fibrofatty tissue, dyspigmentation, and scarring where involution has occurred. Which one of the following would be the most appropriate management? (check one) A. Oral propranolol B. Corticosteroid injection C. Referral for laser therapy D. Referral for surgical excision

Infantile hemangiomas usually appear by 4 weeks of age and stop growing by 5 months of age. As many as 70% leave residual skin changes, including telangiectasia, fibrofatty tissue, redundant skin, atrophy, dyspigmentation, and scarring. Systemic corticosteroids were the mainstay of treatment for hemangiomas during infancy until 2008, when the FDA approved oral propranolol for this indication. Intralesional corticosteroids can be effective for small, bulky, well localized lesions in infants. Laser therapy can also be used to treat early lesions or residual telangiectasia. Once involution is complete, however, as is the case with this child, elective surgical excision is the treatment of choice, producing better outcomes.

A 36-month-old male has persistent deficits in social communication and interaction across multiple contexts. He displays restricted and repetitive patterns of behavior, interest, and activities. According to the DSM-5 , which one of the following is the most appropriate diagnosis? (check one) A. Asperger syndrome B. Autism spectrum disorder C. Autistic disorder D. Childhood disintegrative disorder E. Pervasive developmental disorder not otherwise specified

Key diagnostic features of autism spectrum disorder include deficits in social communication and interaction across multiple contexts and restricted, repetitive patterns of behavior, interests, or activities. The DSM-5 , which came out in 2013, created an umbrella diagnosis of autism spectrum disorder to consolidate four previously separate disorders: autistic disorder, Asperger's disorder, childhood disintegrative disorder, and pervasive developmental disorder not otherwise specified. Any individuals with a previous diagnosis of one of these disorders should now be given a diagnosis of autism spectrum disorder.

A 62-year-old male sees you the day after returning from a 4-day cruise. He says he developed a fever and a productive cough on the day before the ship returned to Los Angeles following a trip down the coast of Baja California. He tells you that several other passengers had similar symptoms. The examination is remarkable for tachypnea and you hear crackles in both lungs.This patient's history should raise concerns about infection with which one of the following pathogens? (check one) A. Asian avian influenza A virus B. Coxiella burnetii C. Hantavirus D. Histoplasma capsulatum E. Legionella species

Legionella should be considered as a pathogen for community-acquired pneumonia when the patient has a history of a hotel stay or cruise ship travel within the past couple of weeks. Travel to or residence in Southeast Asia or East Asia is a risk factor for avian influenza, exposure to farm animals or parturient cats is a risk factor for Coxiella burnetii infection, exposure to bird or bat droppings is a risk factor for Histoplasma capsulatum infection, and travel to or residence in desert Southwest states with deer mouse exposure is a risk factor for Hantavirus infection.

A 64-year-old male presents with increasing dyspnea on exertion. He feels well otherwise and has no chronic medical problems. A physical examination is normal. Pulmonary function testing reveals normal spirometry, with no evidence of an obstructive or restrictive pattern. However, his lung carbon monoxide diffusing capacity (DLCO) is low.Based on these results, which one of the following is the most likely diagnosis? (check one) A. Asthma B. Bronchiectasis C. Chronic pulmonary emboli D. COPD E. Pulmonary fibrosis

Low diffusing capacity of the lungs for carbon monoxide (DLCO) with normal spirometry indicates a disease process that disrupts gas transfer in the lungs without causing lung restriction or airflow obstruction. Common causes include chronic pulmonary emboli, heart failure, connective tissue disease with pulmonary involvement, and primary pulmonary hypertension. Asthma, bronchiectasis, COPD, and pulmonary fibrosis are associated with abnormalities on spirometry.

A 58-year-old male comes to your office for a routine health maintenance examination. He has smoked 1 pack of cigarettes per day for the last 35 years. The U.S. Preventive Services Task Force recommends which one of the following lung cancer screening strategies for this patient? (check one) A. A chest radiograph annually B. Low-dose CT annually C. Sputum cytology every 3 years D. Bronchoscopy every 5 years

Lung cancer is the leading cause of cancer-related deaths in the United States and the third most common cause of death overall. Smoking causes approximately 85% of all U.S. lung cancer deaths. Thirty-seven percent of U.S. adults are current or former smokers. While nearly 90% of people diagnosed with lung cancer will die from the disease, early-stage non-small cell lung cancer has a better prognosis and can be treated with surgical resection. The largest randomized, controlled trial of low-dose CT (LDCT) for lung cancer detection, the National Lung Screening Trial, enrolled 50,000 people age 55-74 with at least a 30-pack-year smoking history and showed a reduction in lung cancer mortality of 16% and a reduction in all-cause mortality of 6.7%. Based on this study and several other randomized, controlled trials, the U.S. Preventive Services Task Force has concluded that LDCT has a high sensitivity and an acceptable specificity for the detection of lung cancer in high-risk persons. The other testing modalities listed have not been validated as acceptable screening strategies for lung cancer.

A 22-year-old female with polycystic ovary syndrome comes in to discuss contraception. She has no other health conditions and takes no medications. Her menses are somewhat irregular, occurring every 28-42 days. She also asks about treatment for her mild hirsutism. Which one of the following medications would be most likely to address her need for contraception and also improve her hirsutism? (check one) A. Spironolactone (Aldactone) B. Cyclic progesterone C. Progesterone-only contraceptive pills D. Oral combined hormonal contraceptives E. A levonorgestrel-releasing IUD (Mirena)

Management of polycystic ovary syndrome is typically aimed at addressing patient symptoms, as well as irregular menses and the risk of endometrial hyperplasia. Infertility may become a therapeutic target for women who desire pregnancy at some point in their lives. In this patient, who needs contraception and hopes to address her hirsutism, combined oral contraceptives are most likely to address both concerns. In addition to suppressing ovulation they also suppress gonadotropin and ovarian androgen production. The estrogen component increases hepatic production of sex hormone binding globulin, thus decreasing androgen bioavailability. Progestin-only pills and the levonorgestrel IUD protect against pregnancy but will not improve hirsutism. Cyclic progesterone every 1-3 months can be used to prevent endometrial hyperplasia but will not provide contraception or address hirsutism. Spironolactone is an androgen receptor antagonist that can decrease hair growth, but it will not provide contraception.

A 22-year-old gravida 1 para 0 with a history of homelessness and recent intravenous drug use presents for prenatal care. Mantoux tuberculin testing is performed and produces 20 mm of induration. She had a negative tuberculin test 1 year ago. A chest radiograph is normal. You decide to treat her with isoniazid for latent tuberculosis infection. Which one of the following should also be given to prevent the development of peripheral neuropathy? (check one) A. Folate B. Vitamin B6 C. Vitamin D D. Duloxetine (Cymbalta) E. Gabapentin (Neurontin)

Most pregnant women with a positive tuberculin skin test are asymptomatic and have no evidence of active tuberculosis, and therefore have latent tuberculosis infection. The risk of reactivation of tuberculosis and progression to active disease is the highest in the first 2 years of conversion. In women with a known conversion to a positive PPD in the last 2 years, treatment with isoniazid (INH), 300 mg daily, is recommended starting after the first trimester. Treatment should last 6-9 months. Pregnant women are at an increased risk for peripheral neuropathy when treated with INH. Vitamin B6 supplementation decreases the risk of developing peripheral neuropathy with the use of INH.

A patient with a BMI of 32 kg/m2 has type 2 diabetes that is currently controlled by lifestyle interventions, including moderate-intensity physical activity and healthy low-calorie meals. The patient asks about nonnutritive sweeteners, containing few or no calories.According to the American Diabetes Association, which one of the following would be the most appropriate advice? (check one) A. Sucrose (table sugar) is preferred B. Nonnutritive sweeteners are acceptable to use C. Nonnutritive sweeteners worsen glucose control D. Sucralose-based sweeteners, such as Splenda, should be avoided E. Sweeteners with aspartame, such as Equal, should be avoided

Nonnutritive sweeteners contain few or no calories. According to the American Diabetes Association, nonnutritive sweeteners may be acceptable to use instead of nutritive sweeteners such as sucrose. They should be used in moderation if they are used. The use of nonnutritive sweeteners can help to reduce overall intake of carbohydrates and calories. They do not significantly affect glycemic control. Research is inconsistent regarding the effects of nonnutritive sweeteners on weight loss, but most systematic reviews and meta-analyses demonstrate a benefit. There is no recommendation to avoid sucralose or aspartame in patients with type 2 diabetes. Beverages sweetened with sugar are associated with an increased risk of type 2 diabetes.

A 27-year-old male has been treating his plaque psoriasis with high-potency topical corticosteroids for several years. He comes to your office to discuss other options since the lesions on his trunk and extremities are becoming resistant to this therapy. Which one of the following treatment strategies would be most appropriate? (check one) A. Switch to topical tazarotene (Avage, Tazorac) B. Add topical calcipotriene (Dovonex, Sorilux) C. Begin oral acitretin (Soriatane) D. Begin an oral corticosteroid E. Begin etanercept (Enbrel) injections

Oral corticosteroids are not indicated in the treatment of plaque psoriasis. All of the other options are indicated only if topical treatments fail. Of the options listed, the combination of a topical corticosteroid and topical calcipotriene is considered the most appropriate for this patient. Another option would be to add topical tazarotene to the topical corticosteroid. However, when tazarotene is used as monotherapy it often fails to clear plaques and increases the incidence of skin irritation.

A 9-year-old female presents with a 4-week history of right knee pain with activity. There is no history of trauma or recent illness. Your examination reveals lateral patellar tracking with extension of the knee. Which one of the following is the most likely diagnosis? (check one) A. Patellofemoral pain syndrome B. Osgood-Schlatter disease C. Growing pains D. Patellar tendinopathy E. Sever's disease

Patellofemoral pain syndrome is one of the most common causes of knee pain in children, particularly adolescent girls. Pain beneath the patella is the most common symptom. Squatting, running, and other vigorous activities exacerbate the pain. Walking up and down stairs is a classic cause of the pain, and pain with sitting for an extended period is also common. The physical examination reveals isolated tenderness with palpation at the medial and lateral aspects of the knee, and the grind test is also positive. Osgood-Schlatter disease is seen in skeletally immature patients. Rapid growth of the femur can cause tight musculature in the quadriceps across the knee joint. It typically appears between the ages of 10 and 15, during periods of rapid growth. Pain and tenderness over the tibial tubercle and the distal patellar tendon is the most common presentation. The pain is aggravated by sports participation, but also occurs with normal daily activities and even at rest. Growing pains most often affect the thigh and quadriceps and occur during late afternoon or evening, or wake the patient at night. The joints are not affected. The pain typically goes away by morning, and may sometimes occur the day after vigorous or unusual activity. Patellar tendinopathy is an overuse injury often seen in those who participate in jumping sports such as volleyball, and is also related to frequent stops and starts in football players. It typically causes infrapatellar pain, and findings include extensor mechanism malalignment, weakness of ankle flexors, and tightness of the hamstring, heel cord, and/or quadriceps. Sever's disease is an overuse syndrome most often seen between the ages of 9 and 14, and is related to osteochondrosis at the insertion of the Achilles tendon on the calcaneal tuberosity. It occurs during periods of rapid growth, causes heel pain during and after activity, and is relieved with rest. It is often related to beginning a new sport or the start of a season.

A 20-year-old male who is in college in another state calls to report that he has developed wheezing, oral itching, and a swollen lower lip after kissing his girlfriend. The symptoms reminded him of an allergic reaction to peanuts he had when he was a child, so he self-administered a dose of epinephrine with his auto-injector 15 minutes ago. His itching and wheezing have improved, and he asks what else he should do. What advice should you provide? (check one) A. He should take oral diphenhydramine (Benadryl) now and prednisone for 3 days B. He should go to the nearest emergency department C. He should schedule a comprehensive reevaluation by an allergist D. No further action is needed

Patients with a peanut allergy can have reactions to infinitesimal amounts of peanut protein, including residue on the lips of other people. This patient has successfully interrupted the course of anaphylaxis. Diphenhydramine can help reduce subsequent symptoms, and prednisone is generally given, although its value is unproven. However, the patient is at risk of a biphasic reaction and should go to an emergency department where additional epinephrine and resuscitation facilities are available. The American Academy of Pediatrics guideline recommends that all peanut-allergic patients who require a dose of adrenalin be observed in an emergency department. Patients who have not already had a full allergy evaluation need to see an allergist, but this patient's peanut allergy has been established. Peanut-allergic patients tend to have accidental exposure about once every 5 years in spite of efforts at avoidance.

A 46-year-old male with a prosthetic heart valve requests your advice regarding antibiotic prophylaxis for an upcoming dental procedure. The patient is allergic to penicillin. Which one of the following would be most appropriate? (check one) A. Ciprofloxacin (Cipro) B. Clindamycin (Cleocin) C. Doxycycline D. Rifampin (Rifadin) E. Trimethoprim/sulfamethoxazole (Bactrim)

Patients with an artificial heart valve should be given antibiotic prophylaxis prior to dental procedures to prevent infectious endocarditis. The organisms that most frequently cause infectious endocarditis include Staphylococcus aureus (31%), Streptococcus viridans (17%), coagulase-negative staphylococci (11%), Enterococcus (11%), Streptococcus bovis (7%), and other streptococci (5%). Amoxicillin is the preferred medication for prophylaxis. Clindamycin or azithromycin can be used in patients with a penicillin allergy. If the penicillin allergy is not associated with anaphylaxis, angioedema, or urticaria, then cephalexin would be an appropriate antibiotic choice. Ciprofloxacin, doxycycline, rifampin, and trimethoprim/sulfamethoxazole are not used for bacterial endocarditis prophylaxis.

A 34-year-old female with systemic sclerosis sees you for a follow-up visit. She is afebrile, with a blood pressure of 132/76 mm Hg, a heart rate of 82 beats/min, and an oxygen saturation of 94% on room air. On examination you note that the patient is thin and has fibrotic skin changes proximal to the elbows and knees, and facial tightening. She does not have increasing shortness of breath but does have ongoing chronic musculoskeletal pain. She is currently taking cyclophosphamide prescribed by her rheumatologist. Pulmonary function tests reveal an FVC <50%, consistent with restrictive lung disease. CT of the chest shows ground-glass opacities and honeycombing of the lower lobes of the lungs. Which one of the following do these findings suggest? (check one) A. Emphysema B. Idiopathic pulmonary fibrosis C. Interstitial lung disease D. Pulmonary edema E. Sarcoidosis

Patients with systemic sclerosis (SS) in its final stages often develop a restrictive lung disease (SOR C). Interstitial lung disease and pulmonary artery hypertension are common. While the restrictive pattern is similar to idiopathic pulmonary fibrosis, this condition is characteristic of SS and is not idiopathic. Emphysema presents with an obstructive pattern on pulmonary function tests. Pulmonary edema can develop from cardiac malfunction and heart failure, but it is not present in this patient. Sarcoidosis is not related to SS. There is a 10-year mortality of 42% in patients with SS who have an FVC <50%. Cyclophosphamide may be helpful in some cases to improve lung function, decrease dyspnea, and improve the patient's quality of life (SOR B).

Additional workup or referral to an endocrinologist for evaluation of precocious puberty would be indicated in which one of the following patients? (check one) A. A 7-year-old female with some pubic hair B. An 8-year-old female with breast buds C. An 8-year-old male with some pubic hair and axillary odor D. An 8-year-old male with penile enlargement E. A 10-year-old female who has recently begun having menses

Penile enlargement in an 8-year-old male is a sign of precocious puberty. Isolated sparse pubic and axillary hair growth and axillary odor is referred to as premature adrenarche, and represents high levels of dehydroepiandrosterone rather than activation of the hypothalamic-pituitary-gonadal axis that leads to puberty. The isolated findings of premature adrenarche are generally considered benign. An 8-year-old with breast buds and a 10-year-old with menarche are within the normal range of expected pubertal development. Penile enlargement typically represents full activation of the hypothalamic-pituitary-gonadal axis and warrants endocrinologic evaluation in boys younger than 9 years of age.

A 29-year-old male presents to your office with a 2-week history of anal pain and bright red blood on his stool with bowel movements. He says he typically has bowel movements every 3-5 days and his stool is usually hard. He has not noted any purulent drainage or perianal masses. Which one of the following is the most likely diagnosis? (check one) A. Anal fistula B. Anal fissure C. Internal hemorrhoids D. External hemorrhoids E. Perirectal abscess

Posterior midline fissures cause pain during and after defecation. Most are caused by the passage of hard stool and when stretched cause bleeding. Conservative therapy consisting of bulk agents and stool softeners usually allows these to heal. Internal hemorrhoids can cause bleeding with the passage of stool but are typically painless. External hemorrhoids can bleed with trauma but typically cause pain with thrombosis, independent of bowel movements. Anal fistulas and perirectal abscesses may intermittently drain purulent material. Abscesses can cause continuous pain, and a perianal mass may be noted on examination.

A 43-year-old female presents with an 8-month history of posttraumatic stress disorder following a motor vehicle accident that severely injured her and a friend. She has had a positive response to counseling and SSRI treatment but continues to have sleep disturbances and nightmares. Which one of the following medications is most likely to decrease the frequency of her nightmares? (check one) A. Clonazepam (Klonopin) B. Divalproex (Depakote) C. Prazosin (Minipress) D. Propranolol

Prazosin is an α-adrenergic receptor antagonist and is recommended for the treatment of nightmares in posttraumatic stress disorder (SOR A). It is thought to reduce sympathetic outflow in the brain. Although clonidine may be tried, evidence of its effectiveness is sparse (SOR C). Clonazepam, propranolol, and divalproex have not been recommended.

Which one of the following community health programs best fits the definition of secondary prevention? (check one) A. An antismoking education program at a local middle school B. Blood pressure screening at a local church C. A condom distribution program D. Screening patients with diabetes mellitus for microalbuminuria

Prevention traditionally has been divided into three categories: primary, secondary, and tertiary. Primary prevention targets individuals who may be at risk to develop a medical condition and intervenes to prevent the onset of that condition. Examples include childhood vaccination programs, water fluoridation, antismoking programs, and education about safe sex. Secondary prevention targets individuals who have developed an asymptomatic disease and institutes treatment to prevent complications. Examples include routine Papanicolaou tests and screening for hypertension, diabetes mellitus, or hyperlipidemia. Tertiary prevention targets individuals with a known disease, with the goal of limiting or preventing future complications. Examples include screening patients with diabetes for microalbuminuria, rigorous treatment of diabetes mellitus, and post-myocardial infarction prophylaxis with β-blockers and aspirin.

A 34-year-old female presents to your office after she was bitten on the hand by a neighbor's cat. The patient has no allergies and has been in good health. You decide to treat the patient with a prophylactic antibiotic. Which one of the following would be the antibiotic of choice? (check one) A. Amoxicillin/clavulanate (Augmentin) B. Azithromycin (Zithromax) C. Cephalexin (Keflex) D. Clindamycin (Cleocin) E. Metronidazole (Flagyl)

Prophylactic antibiotics should be given for all closed-fist injuries unless the skin has not been penetrated, and for puncture wounds caused by cat bites. The antibiotic should have both aerobic and anaerobic activity and include Pasteurella coverage for animal bites and Eikenella coverage for human bites. Suggested regimens include amoxicillin/clavulanate. If the patient is allergic to penicillin, clindamycin pluslevofloxacin or moxifloxacin, which has anaerobic coverage, can be used. Azithromycin, cephalexin, and metronidazole are not first-line antibiotics following a cat bite. Ref: Bystritsky R, Chambers H: Cellulitis and soft tissue infections. Ann Intern Med 2018;168(3):ITC17-ITC32.

A 48-year-old female presents to the emergency department with chest pain. The evaluation, including CT angiography, reveals a pulmonary embolus.Which one of the following initial findings would be the strongest indication for thrombolytic therapy? (check one) A. Elevated troponin B. Hypotension C. Hypoxia D. Bilateral pulmonary emboli E. Right ventricular dysfunction on echocardiography

Recent guidelines have suggested that hypotension (a systolic blood pressure <90 mm Hg or a diastolic blood pressure <60 mm Hg, for 15 minutes or longer) should be treated with thrombolysis in patients who are not at high risk for bleeding. Patients who have other indicators of cardiopulmonary impairment without signs of hypotension should be given anticoagulation therapy and aggressive supportive care, but should not be treated with thrombolytic therapy. If the patient's condition continues to deteriorate as evidenced by the development of hypotension or other clinical indicators of cardiopulmonary compromise, thrombolysis may be considered.

A 32-year-old obese female from Saudi Arabia presents with muscle aches. Her clothing limits sun exposure and you decide to test her for vitamin D deficiency. Which one of the following is the most appropriate test for this condition? (check one) A. Calcium B. Alkaline phosphatase C. 25-hydroxyvitamin D D. 1,25-dihydroxyvitamin D E. Parathyroid hormone

Recommendations to screen for vitamin D deficiency apply only to patients at risk and not to the general population. This patient's obesity and her clothing style, which limits sun exposure to the skin, puts her at increased risk. Additionally, this patient's muscle aches may be a symptom of vitamin D deficiency. The recommended test for this condition is a 25-hydroxyvitamin D level. A 1,25-dihydroxyvitamin D level is recommended to monitor, not diagnose, certain conditions. Parathyroid hormone, calcium, and alkaline phosphatase levels are poor indicators of vitamin D status.

Beyond short-term pain relief, local corticosteroid injection provides the best long-term improvement for which one of the following? (check one) A. Greater trochanteric bursitis B. Knee osteoarthritis C. Lateral epicondylitis D. Subacromial impingement syndrome E. Trigger finger

Reported cure rates for trigger finger after corticosteroid injection range from 54% to 86%. Corticosteroid injection for the other conditions listed results in temporary pain relief, but the underlying conditions are not improved by the injection.

A 42-year-old female with diabetes mellitus comes to your office because of recurrent yeast infections. She is taking numerous agents in an attempt to lower her glucose level. Which one of the following classes of antidiabetic agents is associated with an increased risk for candidiasis? (check one) A. Biguanides such as metformin (Glucophage) B. DPP-4 inhibitors such as sitagliptin (Januvia) C. SGLT2 inhibitors such as empagliflozin (Jardiance) D. GLP-1 receptor agonists such as liraglutide (Victoza) E. Sulfonylureas such as glipizide (Glucotrol)

SGLT2 inhibitors are known to cause an increased risk of yeast vaginitis because their mechanism of action involves blocking renal uptake of glucose, which results in an increase in glucosuria (SOR A). Common side effects of metformin include gastrointestinal upset. DPP-4 inhibitors have very few side effects.GLP-1 receptor agonists typically cause nausea and early satiety and weight loss. Sulfonylureas are associated with weight gain and hypoglycemia.

A 55-year-old male veteran sees you for a routine health maintenance visit. He is up to date on recommended immunizations. His father was diagnosed with colon cancer at age 70 and his family history is otherwise negative. The patient underwent a colonoscopy at age 50 and has a copy of his pathology results, which describe a single hyperplastic polyp taken from the rectum.Which one of the following would be the most appropriate screening for colorectal cancer in this patient? (check one) A. Annual fecal immunochemical testing (FIT) starting now B. Repeat colonoscopy now C. Repeat colonoscopy at age 60 D. Referral to a gastroenterologist for further management

Screening for colorectal cancer (CRC) is recommended for average-risk individuals beginning at age 50 (SOR A). Individuals at higher risk include those with a personal history of adenomatous polyps, CRC, inflammatory bowel disease, genetic cancer syndromes, or a family history of either adenomatous polyps or CRC. Patients with a first degree relative with CRC or adenomatous polyps discovered before age 60, or two or more first degree relatives at any age with CRC or advanced adenoma, should undergo colonoscopy screening starting at age 40 or 10 years before the youngest age a family member was diagnosed, whichever comes first. The maximum surveillance interval for these patients is 5 years (SOR C). Patients with a single first degree relative diagnosed at age 60 or older, and patients with two affected second degree relatives, should undergo screening starting at age 40 by any recommended method, and at the same intervals for average-risk individuals (SOR C). Patients with small, distal hyperplastic polyps are considered to have a normal colonoscopy (SOR C). There is no need for referral to a gastroenterologist or interval fecal immunochemical testing (FIT) following an adequate colonoscopy.

A 52-year-old male sees you for the first time for a health maintenance visit. He previously tested positive on a hepatitis C antibody test performed by another physician. He seeks your opinion regarding follow-up testing.Which one of the following tests would you suggest at this point? (check one) A. α-Fetoprotein B. Hepatitis B antigen C. Hepatitis C RNA D. Ultrasonography of the liver E. A liver biopsy

Screening for hepatitis C virus (HCV) with an anti-HCV antibody test is recommended for all adults at high risk of infection, and one-time screening is recommended in adults born between 1945 and 1965. If the anti-HCV antibody test result is positive, current infection should be confirmed with a qualitative HCV RNA test.

A 32-year-old female presents with a 4-month history of nasal drainage, congestion, and loss of her sense of smell. She reports having a cold about 4 months ago that never resolved. On examination the nasal turbinates are swollen and you note mucopurulent drainage on the right. Which one of the following is the most likely cause of her symptoms? (check one) A. Chronic rhinosinusitis B. Granulomatosis with polyangiitis (Wegener's granulomatosis) C. Nasal polyposis D. Sarcoidosis E. Seasonal allergic rhinitis

The American Academy of Otolaryngology defines chronic rhinosinusitis as the presence of two of four cardinal symptoms, which include nasal drainage, nasal obstruction, facial pain or pressure, and hyposmia or anosmia, along with objective signs on examination or radiographic studies. This patient has three cardinal symptoms of chronic rhinosinusitis and objective evidence on the physical examination. No nasal polyps were seen on the examination. Granulomatosis with polyangiitis and sarcoidosis can both present similarly but are uncommon causes of chronic rhinosinusitis. Allergic rhinitis can be associated with chronic rhinosinusitis but would also present with allergic symptoms.

A patient is admitted to the hospital with severe acute pancreatitis, based on diagnostic criteria for severity. After appropriate intravenous hydration, which one of the following is associated with shorter hospital stays and lower mortality? (check one) A. Parenteral nutrition B. Nothing by mouth until the pain has resolved C. Clear liquids by mouth after 48 hours D. Bolus nasogastric enteral nutrition E. Continuous nasogastric enteral nutrition

The American College of Gastroenterology recommends that patients with severe acute pancreatitis receive enteral nutrition. Enteral feedings help prevent infectious complications, such as infected necrosis, by maintaining the gut mucosal barrier and preventing translocation of bacteria that may seed pancreatic necrosis. Currently, continuous enteral feeding is preferred over bolus feeding. A meta-analysis has shown that continuous nasogastric enteral feeding started in the first 48 hours decreases mortality and the length of hospital stay. Total parenteral nutrition is not recommended because of infectious and line-related complications. It should be avoided unless the enteral route cannot be used.

A 25-year-old gravida 1 para 1 presents for insertion of a levonorgestrel-releasing intrauterine device (Mirena). She is on the last day of her menses, which began 5 days ago. A urine pregnancy test in the office is negative. You insert the device without complications and she asks how long she needs to use backup contraception. Which one of the following would be the most appropriate advice? (check one) A. Backup contraception is not necessary B. She should use backup contraception for the next 48 hours C. She should use backup contraception for the next 7 days D. She should use backup contraception for the next 14 days E. She should use backup contraception for the next month

The Centers for Disease Control and Prevention (CDC) provides specific recommendations for backup contraception after IUD insertion. According to the CDC guidelines, this patient does not need to use backup contraception if her IUD is inserted today because it was inserted within 7 days after menstrual bleeding started. If the levonorgestrel IUD is inserted more than 7 days after menstrual bleeding starts, the patient needs to abstain from sexual intercourse or use additional contraceptive protection for the next7 days.

An 85-year-old female presents to your office with her daughter to discuss the benefits and risks of oral anticoagulation, and to address her fall risk. She has chronic atrial fibrillation and mild cognitive impairment, and meets the criteria for frailty. She lives with the daughter and uses a walker but is independent for basic activities of daily living (ADLs) such as feeding, bathing, and toileting. She does need assistance with paying bills and other instrumental ADLs. Her quality of life is good overall and she enjoys interacting with friends and family. She has been falling about once a month but has not sustained a serious injury. Her blood pressure is 140/70 mm Hg. She does not use tobacco, alcohol, or illicit drugs. She takes alendronate (Fosamax), 70 mg weekly, and hydrochlorothiazide, 12.5 mg daily. She has been on warfarin (Coumadin) for about 3 years for the atrial fibrillation, with an INR of 2-3. She has not had a stroke. Laboratory findings are significant for a serum creatinine level of 0.9 mg/dL (N 0.6-1.2) and normal liver enzyme levels. You engage in shared decision making with the patient and her daughter regarding oral anticoagulation. Which one of the following would you advise them about the risks and benefits of oral anticoagulation for this patient? (check one) A. The benefits outweigh the risks B. The risks outweigh the benefits because of her age C. The risks outweigh the benefits because of her frequency of falls D. The risks outweigh the benefits because of her cognitive impairment E. The risks outweigh the benefits because of her frailty

The European Society of Cardiology 2016 Guidelines for Atrial Fibrillation state that the benefits of oral anticoagulation outweigh the risks in the majority of patients with atrial fibrillation who meet CHA2DS2-VASc criteria for oral anticoagulation. This includes the elderly and patients with cognitive impairment, frailty, or frequent falling. Oral anticoagulation is superior to aspirin for the prevention of stroke, while the bleeding risk with aspirin is not different than that of oral anticoagulation. Use of the CHA2DS2-VASc criteria significantly increases the number of patients eligible for anticoagulation therapy compared with the CHADS2 scoring system. If there is concern about bleeding risk, particularly in patients older than 65 years of age, the HAS-BLED scoring system has been well validated, with a score of 3 or more indicating that a patient has a high likelihood of hemorrhage. This patient's HAS-BLED score is 1 (age) and her estimated risk of major bleeding with 1 year of anticoagulation is 1.88%-3.3%. Her adjusted stroke risk is high (4.8% per year), as she has a CHA2DS2-VASc score of 4 (age ≥75, female, history of hypertension).

A 67-year-old male is admitted to the intensive-care unit with complications of sarcoma of the leg, including multiple pulmonary emboli. He most likely will require an urgent above-the-knee amputation. A spiritual assessment in this patient (check one) A. should be delegated to the hospital chaplain B. can be performed by the physician C. is not appropriate if the patient has not listed a religion in his demographics D. should be postponed until the patient is in less critical condition

The Joint Commission for Hospital Accreditation now requires a patient spiritual assessment upon hospital admission. Using the FICA Spiritual History Tool or HOPE questions for making the spiritual assessment is appropriate for the physician. HOPE is a mnemonic for sources of Hope, Organized religion, Personal spirituality and practices, and Effects on medical care and end-of-life issues. The FICA tool includes questions in the categories of Faith and beliefs, Importance, Community, and how to Address these issues when providing care. It is very appropriate for a physician to conduct a spiritual assessment in older, hospitalized patients with critical or terminal illnesses. Some patients may consider themselves spiritual but not necessarily religious.

A 50-year-old male sees you for a health maintenance visit. He has a 20-year history of smoking 1 pack of cigarettes per day but his history is otherwise unremarkable. He is not aware of any disease that runs in his family, including diabetes mellitus. He has not visited a physician for the past 5 years. A physical examination reveals a BMI of 28.2 kg/m², normal blood pressure, and no other significant findings. Laboratory testing reveals a fasting plasma glucose level of 107 mg/dL. According to the U.S. Preventive Services Task Force, which one of the following would be most appropriate at this point? (check one) A. No further diabetes screening unless his BMI increases to ≥30 kg/m² B. A repeat fasting plasma glucose level in 1-2 weeks C. A repeat fasting plasma glucose level in 1 year D. A repeat fasting plasma glucose level in 3 years E. Treatment with metformin (Glucophage)

The U.S. Preventive Services Task Force (USPSTF) recommends screening for abnormal blood glucose levels as part of a cardiovascular disease risk assessment for adults who are 40-70 years of age and who are overweight (BMI 25.0-29.9 kg/m²) or obese (BMI ≥30.0 kg/m²). Since his fasting blood glucose result was in the range consistent with impaired fasting glucose (100-125 mg/dL), the USPSTF recommends confirming the diagnosis of the abnormal glucose level soon by performing the same test on a different day. Appropriate treatment should begin once the diagnosis is confirmed.

A 20-year-old female presents to your office with questions about her contraceptive method. She has been using a combined oral contraceptive pill for the past 2 years without any complications. She has learned that several of her friends recently switched to an IUD. She is concerned about the efficacy of her current method and asks about the failure rate.You tell her that with typical use, the annual failure rate of a combined oral contraceptive pill is (check one) A. 0.2% B. 2% C. 9% D. 18% E. 22%

The annual failure rate of combined oral contraceptive pills with typical use is 9%. Typical failure rates for other contraceptive methods are 0.2% for the levonorgestrel IUD, 6% for injectable progestin, 18% for male condoms, and 22% for the withdrawal method.

An 85-year-old female is admitted to the hospital for surgery and develops confusion postoperatively. The patient is a widow and lived independently prior to admission. Her daughter says the confusion is atypical for her mother. She does not have a history of memory loss, forgetfulness, or confusion prior to admission. Which one of the following cognitive assessment tests should be used to assess her acute change in mental status? (check one) A. Confusion Assessment Method (CAM) B. Mini-Cog C. Mini-Mental State Examination (MMSE) D. Montreal Cognitive Assessment test (MoCA) E. Saint Louis University Mental Status exam (SLUMS)

The patient is experiencing an acute cognitive change from baseline, indicating possible delirium. The Confusion Assessment Method (CAM) is a delirium diagnosis tool useful for evaluating acute cognitive changes. The other tests listed, including the Mini-Mental State Examination, Mini-Cog, Montreal Cognitive Assessment, and Saint Louis Mental Status exam, test chronic baseline cognitive function and are not designed to test for acute changes

A 26-year-old female has a 4-month history of continuous right-sided headache. The headache is associated with tearing and nasal congestion only on the right, and has not responded to over-the-counter analgesics. The patient went to the emergency department a few nights ago because of the pain, and CT of the head at that visit was normal. On examination you note conjunctival injection on the right. Findings are otherwise normal.Which one of the following would be the most appropriate treatment at this time? (check one) A. Sumatriptan (Imitrex) B. Amitriptyline C. Indomethacin D. Topiramate (Topamax)

There are several types of chronic headaches, and they often respond to different treatments. Migraine is very prevalent and is characterized by headaches that are periodic, often unilateral, and frequently pulsatile. Migraine is familial and typically starts in childhood, adolescence, or young adulthood, and the headaches decrease in frequency over time. Some are associated with aura, which causes visual disturbances. In mild cases, over-the-counter medications may control symptoms. For most patients, however, treatment to control the attack can include triptans such as sumatriptan, and/or ergot alkaloids such as ergotamine. Treatment to prevent attacks may also be appropriate, and could include a β-blocker, antiepileptic drugs, or amitriptyline. Tension headaches are usually bilateral and are typically described as dull or aching, but patients often describe tightness or pressure. They are not associated with symptoms such as throbbing, nausea, or photophobia. Tension headaches are more frequent than migraine but patients often treat them at home without seeking medical treatment. Frequent or persistent tension headaches can be treated with several drugs used for anxiety or depression, including amitriptyline. Stronger analgesics and ergotamine are not helpful. Cluster headache is another type of chronic headache. This occurs most frequently in adult males, and often occurs over a period which may extend over many weeks, with repeated episodes or clusters. It most often occurs at night, and may recur several times during the night. The headache is unilateral and is associated with orbital pain and vasomotor phenomenon such as blocked nasal passages, rhinorrhea, conjunctival injection, and miosis. The headache can be treated with inhalation of 100% oxygen, and the headache cycle can be terminated with verapamil. Ergotamine or sumatriptan can be used at night to prevent attacks. There are also variants of cluster headaches, including chronic paroxysmal hemicrania, which resembles cluster headache but has some important differences. Like cluster headaches, these headaches are unilateral and accompanied by conjunctival hyperemia and rhinorrhea. However, these headaches are more frequent in women, and the paroxysms occur many times each day. This type of headache falls into a group of headaches that have been labeled indomethacin-responsive headaches because they respond dramatically to indomethacin.

A pregnant 20-year-old female is diagnosed with a Chlamydia infection. She is treated and the infection is cured, but she tests positive again at a follow-up visit. In this situation, contacting her sexual partners is considered a reasonable breach of confidentiality because (check one) A. there is a public health risk B. there is a risk to the fetus C. the patient's Chlamydia infection has become resistant to antibiotics D. there is a risk of pelvic inflammatory disease

There are three situations when a breach of confidentiality is justified: abuse of a vulnerable person (child or elderly), a public health risk (communicable disease), or substantial danger to the patient or others. While Chlamydia is not usually considered life-threatening or dangerous, it is communicable. Contacting sexual partners to notify and treat them to stem the spread of disease is recommended.

A 69-year-old female presents with postmenopausal bleeding. You consider whether to begin your evaluation with transvaginal ultrasonography to assess the thickness of her endometrium. In evaluating the usefulness of this test to either support or exclude a diagnosis of endometrial cancer, the most useful statistic is the (check one) A. likelihood ratio B. number needed to treat C. prevalence D. incidence E. relative risk

There has been a large increase in the number of diagnostic tests available over the past 20 years. Although tests may aid in supporting or excluding a diagnosis, they are associated with expense and the potential for harm. In addition, the characteristics of a particular test and how the results will affect management and outcomes must be considered. Clinically useful statistics for evaluating diagnostic tests include the positive predictive value, negative predictive value, and likelihood ratio. The likelihood ratio indicates how a positive or negative test correlates with the likelihood of disease. Ratios greater than 5-10 greatly increase the likelihood of disease, and those less than 0.1-0.2 greatly decrease it. In the example given, if the patient's endometrial stripe is >25 mm, the likelihood ratio is 15.2 and her post-test probability of endometrial cancer is 63%. However, if it is ≤4 mm, the likelihood ratio is 0.02 and her post-test probability of endometrial cancer is 0.2%. The number needed to treat is useful for evaluating data regarding treatments, not diagnosis. Prevalence is the existence of a disease in the current population, and incidence describes the occurrence of new cases of disease in a population over a defined time period. The relative risk is the risk of an event in the experimental group versus the control group in a clinical trial.

A 2-month-old male is brought to your office for a well child check. The parents tell you that he regurgitates with every feeding, and they are getting very frustrated. He takes about 4-5 ounces of formula every 4 hours. With every feeding he regurgitates a large amount of formula. They have tried frequent burping and elevating his head during and after feeding, but he continues to regurgitate. He does not appear to be uncomfortable and overall is a happy baby. He is growing well and a complete examination is normal. Which one of the following is the most appropriate management for this patient? (check one) A. Reassurance B. Metoclopramide (Reglan) C. Omeprazole (Prilosec) D. Ranitidine (Zantac) E. Abdominal ultrasonography

This child has gastroesophageal reflux. This is a normal physiologic process that occurs in infants. Most reflux events are caused by transient lower esophageal sphincter relaxation that is triggered by postprandial gastric distention. This relaxation can continue into childhood, but with growth and an upright eating position it generally improves. Reflux in infants can be treated by implementing body position changes while awake, lower volume feedings if they are overfed, thickening agents, and antiregurgitant formula. It is recommended to avoid the use of medication in infants that have regurgitation that is effortless, painless, and not affecting growth. When the infant is not growing well or appears to be in pain, it would be appropriate to initiate pharmacotherapy with an acid-suppressing medication. Abdominal ultrasonography would be indicated if there were forceful vomiting and concerns about possible pyloric stenosis.

An 8-year-old male is brought to your office because of a rash on his arms and legs that has been present for the last several weeks. It seems to be spreading gradually, according to his parents. No other family members have similar symptoms. A physical examination reveals excoriated erythematous papules on both forearms and both lower legs that drain a small amount of serous fluid. The child says that these are itchy. There are no signs of deeper inflammation and no lesions are present on the scalp, hands, thorax, or groin. Which one of the following is the most likely cause of this problem? (check one) A. Bedbugs B. Brown recluse spiders C. Mites/chiggers D. Scabies E. Ticks

This child's presentation appears most consistent with bites from an insect. Having multiple exposures on skin often not covered by clothing would be typical of household fleas or bedbugs. Tick bites are typically identified by the presence of an actively feeding insect or a single papular lesion. Similarly, brown recluse spider bites would not be expected to be multiple or recurrent. A chigger is the larval form of a mite, which is an eight-legged arthropod. The larval form has only six legs, and tends to crawl into spaces near constricted clothing and cause welts from their bites along the neckline, waistline, sock line, or more rarely on the genitals. A scabies infestation often presents as an eczematous rash in semi-protected folds of skin such as the web spaces of fingers, the umbilicus, the axillae, or the genital region.

A 45-year-old male with a 30-pack-year smoking history reports a chronic cough with a small amount of phlegm production and dyspnea with strenuous exercise. You order spirometry, which shows a pre- and postbronchodilator FEV1/FVC ratio of 0.6 and an FEV1 of 85% of predicted.Which one of the following agents would be the best initial pharmacologic management? (check one) A. An inhaled corticosteroid B. A short-acting anticholinergic C. A long-acting anticholinergic D. A long-acting β2-agonist E. Theophylline

This patient has COPD and is in a risk category of A (low risk, fewer symptoms) based on the Global Initiative for Chronic Obstructive Lung Disease (GOLD) combined assessment of COPD. As a result, either a short-acting anticholinergic or a short-acting β2-agonist should be selected as the initial pharmacologic management. Long-acting β2-agonists or long-acting anticholinergics are indicated for patients with a GOLD combined assessment category of B or worse. Long-acting inhaled corticosteroids are indicated for patients with a GOLD combined assessment category of C or worse. Due to its narrow therapeutic window, modest benefit, and need for monitoring, theophylline is not recommended as an initial agent and should be considered as an alternative only for patients with severe refractory symptoms.

A 62-year-old female presents with numbness and tingling in her feet. She first noticed tingling in the toes of her right foot several months ago; it is now present in both feet and is causing numbness. She has not experienced any weakness, or any changes in vision, speech, or memory. Her medical history includes hypertension controlled by lisinopril (Prinivil, Zestril), 20 mg daily, and she also takes aspirin, 81 mg daily. She drinks a glass of wine nightly and does not smoke. She does not have a family history of neurologic disorders.On examination she has symmetric decreased sensation to light touch and vibration in her feet. Reflexes and strength are intact bilaterally. Laboratory findings include a normal CBC and normal TSH and vitamin B12 levels. Her erythrocyte sedimentation rate is 32 mm/hr (N 0-20). A comprehensive metabolic panel is normal except for a total protein level of 8.5 g/dL (N 6.0-8.3).Which one of the following tests would be most useful for making a diagnosis? (check one) A. An angiotensin converting enzyme level B. Serum protein electrophoresis C. A chest radiograph D. A lumbar puncture with cerebrospinal fluid analysis E. MRI of the lumbar spine

This patient has a peripheral neuropathy. A review of the patient's history and specific laboratory testing was performed to evaluate for the most common treatable causes of peripheral neuropathy, which include diabetes mellitus, hypothyroidism, and nutritional deficiencies. Additional causes of peripheral neuropathy include chronic liver disease and renal disease. It is important to consider medications as a possible cause, including amiodarone, digoxin, nitrofurantoin, and statins. Excessive alcohol use is another important consideration. In this patient, the mildly elevated total protein and erythrocyte sedimentation rate, which suggest a monoclonal gammopathy such as MGUS (monoclonal gammopathy of unknown significance) or multiple myeloma, should direct her workup. Serum protein electrophoresis is indicated to assess for this. Other less common causes of peripheral neuropathy include carcinoma causing a paraneoplastic syndrome, lymphoma, sarcoidosis, AIDS, and genetic disorders such as Charcot-Marie-Tooth disease. Approximately 25% of patients with peripheral neuropathy have no clearly defined cause after a thorough evaluation and are diagnosed with idiopathic polyneuropathy. MRI of the lumbar spine can identify central lesions causing spinal cord or nerve root compression but is not indicated in the evaluation of peripheral neuropathy. Serum angiotensin converting enzyme levels and a chest radiograph can assist in the diagnosis of sarcoidosis, which can cause peripheral neuropathy but is less likely in this patient. Cerebrospinal fluid analysis is important in assessing for chronic inflammatory demyelinating polyradiculoneuropathy, a more rare cause of peripheral neuropathy.

An 85-year-old male nursing home resident with a past history of a stroke has developed a pressure ulcer over his right greater trochanter. The ulcer is 2 cm in size and is noted to be shallow with a reddish-pink wound base. There is no evidence of secondary infection.Which one of the following would be best for cleansing the wound? (check one) A. Tap water B. Aluminum acetate (Burow's solution) C. Hydrogen peroxide D. Povidone/iodine solution (Betadine) E. Sodium hypochlorite (Dakin's solution)

This patient has a stage 2 pressure ulcer. It is recommended that pressure ulcers not be cleaned with povidone/iodine, Dakin's solution, hydrogen peroxide, wet-to-dry dressings, or any solutions that may impede granulation tissue formation. These sites should be cleaned with either saline or tap water and covered with hydrocolloid, foam, or another nonadherent dressing that promotes a moist environment.

While on call for your group practice you are called to admit a 23-year-old female with a history of sickle cell disease who presented to the emergency department with chest pain, a cough, and shortness of breath. She has no history of recent hospitalization.Physical FindingsBlood pressure ..................................... 176/86 mm HgPulse.........................................................103 beats/minRespiratory rate ..................................... 20/minTemperature............................................ 37.8°C (100.0°F)Oxygen saturation................................... 89% on room air A chest radiograph shows consolidation in the right lower lobe. In addition to oxygen, intravenous fluids, an intravenous third-generation cephalosporin, and pain management, which one of the following is important to include in the patient's treatment plan? (check one) A. Azithromycin (Zithromax) B. Daptomycin (Cubicin) C. Ertapenem (Invanz) D. Gentamicin E. Vancomycin (Vancocin)

This patient has acute chest syndrome (ACS), a serious vaso-occlusive complication of sickle cell disease (SCD). Its cause may be multifactorial, but infections are common and antimicrobials are indicated. However, the clinical course of ACS is significantly different from infectious pneumonia in patients without SCD, due to the damaged microvasculature that occurs in ACS. Studies have shown that atypical pathogens predominate in ACS and it is therefore important to treat all patients with ACS with antibiotics that cover Mycoplasma and Chlamydophila . Viral infections are also common, especially in children with ACS. Other possible pathogens include Staphylococcus aureus , Streptococcus pneumoniae , and Haemophilus influenzae . Therefore, the use of a third-generation cephalosporin along with azithromycin is the recommended antibiotic coverage. In addition to antimicrobials, treatment includes supportive care with supplemental oxygen, intravenous fluids, pain control, and incentive spirometry. Depending on the degree of anemia seen, a simple blood transfusion or exchange transfusion is often indicated as well. Consultation with a hematologist is recommended in the care of patients with ACS. Even with appropriate care, mortality rates in ACS are as high as 3%.

A 67-year-old male presents to your office for evaluation of chronic redness, flaking, and discomfort of his eyelids. Additionally, his eyes feel irritated, dry, and sandpapery at times. He has had difficulties with these symptoms on and off throughout his life but they have worsened lately. He has not had any vision changes and does not wear contact lenses.On examination his eyelids appear red and mildly swollen with yellow crusting at the bases of the eyelashes. You note bilateral mild conjunctival injection. Visual acuity is intact, as are pupil reactions and extraocular movements.Which one of the following treatments is appropriate first-line therapy for this condition? (check one) A. Warm compresses and gentle cleansing with a mild shampoo B. Sodium sulfacetamide eye drops C. Topical betamethasone D. Oral acyclovir (Zovirax) E. Oral cephalexin (Keflex)

This patient has blepharitis, a chronic inflammation of the eyelids. Seborrhea is a common cause in older adults. In younger patients including children, colonization with Staphylococcus may be a contributing factor. Meibomian gland dysfunction is often part of this condition, contributing to a reduced quality of tear films, which leads to dry eyes and irritation. Other diagnoses to consider in this patient include conjunctivitis, preseptal cellulitis, and Sjögren's syndrome. Conjunctivitis typically involves the conjunctiva and an eye discharge but less involvement of the eyelids is present. Cellulitis is an acute rather than chronic condition and involves more pain and swelling. Sjögren's syndrome causes dry eye but not inflammatory changes of the lid. The initial treatment of blepharitis consists of lid hygiene using warm compresses to remove dried secretions and debris. Mild shampoo can help in this process and aid in keeping the bacterial colonization load down. In severe or recalcitrant cases a topical antibiotic ointment may be applied to the lids. Oral antibiotics can be considered for more severe cases.

A 77-year-old female with widely metastatic lung cancer is seen in her home with increased confusion, hallucinations, and combative behavior for the past 2 days. She has not slept and has a very poor appetite.Which one of the following should be the first step in the management of this patient? (check one) A. Place her in gentle restraints for safet B. Determine and treat the underlying cause of the behavior change C. Start diphenhydramine (Benadryl) at bedtime for sleep D. Start mirtazapine (Remeron) at bedtime for sleep and appetite E. Start lorazepam (Ativan) as needed for agitation

This patient has hyperactive delirium. The first step in management is to determine and treat the underlying cause if possible. There are multiple causes of delirium such as medications, infections, metabolic abnormalities, and underlying diseases. The first step in treatment is behavioral management with strategies to orient the patient. Haloperidol or antipsychotics may be used if the patient is at risk of harm. Lorazepam and anticholinergics should both be avoided, as they can worsen delirium. Restraints can also worsen the agitation and should not be used. Mirtazapine is an antidepressant and is not used in the treatment of delirium.

A 2-year-old female is brought to your office because of a round lesion on her lip that appeared 2 days ago. Her temperature and all vital signs are normal. She has no past medical history and takes no medications. Further history reveals that she was playing with a toy trumpet in a busy store a few days before the lesion appeared. A physical examination reveals a 1-cm round lesion with crusting, and no other skin abnormalities. Which one of the following would be the best treatment at this time? (check one) A. Bacitracin B. Mupirocin (Bactroban) C. Neomycin D. Cephalexin (Keflex) E. Clindamycin (Cleocin)

This patient has physical findings and a history consistent with impetigo, a skin infection caused by Staphylococcus aureus and/or Streptococcus pyogenes . Since she has only one lesion, systemic antibiotics are not required as they would be for a patient with extensive disease or multiple lesions. Although bacitracin and neomycin are commonly used, they are much less effective for impetigo than mupirocin, despite some reports of resistance to mupirocin (level A-1 evidence).

A 42-year-old male hair stylist presents with numbness and tingling of the right fifth finger that has been progressive over the last few months. He is particularly bothered after a long day of prolonged elbow flexion. He has no pain but has noticed some clumsiness and difficulty with fine coordination of his fingers. An examination clearly shows intrinsic muscle weakness.Which one of the following is the most likely diagnosis? (check one) A. Anterior interosseous nerve syndrome B. Carpal tunnel syndrome C. Cubital tunnel syndrome D. Pronator syndrome E. Wartenberg syndrome

This patient has signs and symptoms of cubital tunnel syndrome, which is the second most common peripheral neuropathy. Symptoms develop because of ulnar nerve compression in the upper extremity, leading to sensory paresthesias in the ulnar digits and intrinsic muscular weakness. Vague motor problems, including poor coordination of the fingers and hand clumsiness, are frequent complaints. Provocative testing includes demonstration of Tinel's sign over the cubital tunnel, and the elbow flexion test with paresthesias elicited over the ulnar nerve. Carpal tunnel syndrome causes paresthesias in the distal median nerve distribution. Wartenberg's syndrome reflects compression of the superficial radial nerve. Pronator syndrome is a proximal median nerve neuropathy, while anterior interosseous nerve syndrome, a rare clinical entity, causes paresis or paralysis of the flexor pollicis longus, and the flexor digitorum profundus of the index and long fingers.

A 47-year-old female with a 10-year history of type 2 diabetes mellitus is concerned about the recent onset of swelling in her legs accompanied by a sudden weight gain of 8 lb. She is also experiencing increased fatigue and shortness of breath with mild exertion. On examination she has a blood pressure of 150/95 mm Hg, which is above her baseline of 130/85 mm Hg. Her lungs are clear to auscultation and a cardiac examination is also normal. She has no hepatosplenomegaly, but her legs are swollen to the level of the mid tibia bilaterally. You are concerned that her symptoms and examination findings maybe related to an underlying renal pathology. To confirm your suspicion, the most appropriate diagnostic test at this time would be (check one) A. a spot urine protein to creatinine ratio B. a 24-hour urine creatinine determination C. renal ultrasonography D. renal enhanced MRI E. renal biopsy

This patient has type 2 diabetes mellitus and presents with new-onset edema in her lower extremities, the most common presenting symptom of nephrotic syndrome (NS). Patients with NS may also report foamy urine, exertional dyspnea or fatigue, and significant fluid-associated weight gain. A 24-hour urine collection for protein (not creatinine) can be used to diagnose proteinuria, but the collection process is cumbersome and the specimen is often collected incorrectly. The protein-to-creatinine ratio from a single urine sample is commonly used to diagnose nephrotic-range proteinuria. The role of a renal biopsy inpatients with NS is controversial and there are no evidence-based guidelines regarding indications for a biopsy. Renal ultrasonography may be appropriate to assess for underlying conditions and/or disease complications if the glomerular filtration rate is reduced. There is no data to support using MRI in the diagnosis and management of nephrotic syndrome.

A home health nurse calls you about a 62-year-old male who is recovering at home several days after spinal surgery. His recovery was going well until he became unable to urinate despite the sensation of needing to do so. His last normal void was about 12 hours ago and felt incomplete. Catheterization produced 900 mL of clear-appearing urine that she will send for urinalysis. His bowel movements have been normal and his need for pain medications has been decreasing. In addition to stopping medications that may be promoting his urinary retention, which one of the following management strategies would be most appropriate? (check one) A. Leaving the indwelling catheter in place for 48-72 hours B. Starting finasteride (Proscar), 5 mg daily C. Starting oxybutynin, 10 mg daily D. Sending the patient to the emergency department

This patient is suffering from acute urinary retention, likely due to mild benign prostatic hyperplasia exacerbated by pain medication and a lack of activity. Acute urinary retention could also possibly be due to irritation of sympathetic and/or parasympathetic nerves near the spine. Placing an indwelling bladder catheter is appropriate. It would also be reasonable, although impractical in the short term, to teach the patient or his caretakers to intermittently catheterize him. The likelihood of a successful return to voiding spontaneously will increase over time. However, the risk of catheter-associated urinary tract infection is estimated to be about 5% per day. Therefore, catheter removal and a trial of spontaneous voiding should be attempted after 48-72 hours. There is good evidence that starting an α-blocking medication such as tamsulosin during the time the catheter is in place will nearly double the success of the trial of spontaneous voiding. Finasteride in isolation is not recommended and oxybutynin would be contraindicated.

A 78-year-old male presents for a routine health maintenance examination and is concerned about a gradual loss in his vision during the past year. He has smoked 1 pack of cigarettes per day for the past 60 years. He has no other medical problems. On Amsler grid testing he notes distorted grid lines.Which one of the following would you recommend for this patient? (check one) A. Watchful waiting B. Avoiding all vitamin supplements C. Treatment to reverse his visual changes D. Smoking cessation to prevent further vision loss

This patient presents with signs and symptoms that suggest age-related macular degeneration. Smoking is a modifiable risk factor and smokers should be counseled to quit (SOR C). The patient should be referred to an ophthalmologist for further evaluation and management. Watchful waiting would not be appropriate. Vitamin supplements with Age-Related Eye Disease (AREDS) and AREDS2 formulations have been shown to delay visual loss in patients with age-related macular degeneration (SOR A). Age-related macular degeneration is not reversible but treatment can delay progression or stabilize the changes (SOR A).

A 48-year-old female is treated appropriately for MRSA bacteremia. An echocardiogram is negative for endocarditis. There are no indwelling devices such as prosthetic heart valves or vascular grafts. Assuming that the patient improves with an excellent response to antibiotics, which one of the following is recommended? (check one) A. No repeat blood cultures B. Repeat blood cultures when the antibiotic course is completed C. Repeat blood cultures when the patient's temperature is ≤37.5°C (99.5°F) D. Repeat blood cultures 2-4 days after the initial set and as needed thereafter E. Repeat blood cultures 2 weeks after the antibiotic course is completed

This patient's MRSA bacteremia is considered uncomplicated due to the effectiveness of the antibiotic therapy and the lack of endocarditis or implanted prostheses such as heart valves. Therefore, the Infectious Diseases Society of America recommends that follow-up cultures of blood samples be obtained 2-4 days after the initial cultures and as needed thereafter to document clearance of bacteremia (SOR A; Quality of Evidence II).

A 43-year-old female is very distressed about symptoms of postprandial fullness and early satiety. She has seen several physicians over the last 6 months but said she was always told "nothing's wrong." You review her most recent workup, including normal blood tests, normal gallbladder testing, and upper endoscopy that shows no abnormalities, including negative testing for Helicobacter pylori . She has tried multiple antacid medications, including omeprazole (Prilosec), lansoprazole (Prevacid), and ranitidine (Zantac), with no success. Which one of the following medications has the best chance of providing this patient with symptom relief? (check one) A. Clonazepam (Klonopin) B. Escitalopram (Lexapro) C. Metoclopramide (Reglan) D. Ondansetron (Zofran) E. Sucralfate (Carafate)

This patient's history fits the diagnosis of functional dyspepsia. Two subtypes of this disorder have been described. The first, epigastric pain syndrome, is described as intermittent pain and burning in the epigastrium. The second, postprandial distress syndrome, is more typical of the symptoms this patient describes: postprandial fullness and early satiety. Although there is considerable benefit from reassurance and "naming" a patient's condition, empiric treatment is also warranted. Patients with epigastric pain syndrome are more likely to respond to proton pump inhibitors or H2-blockers. Patients with predominantly postprandial distress symptoms are more likely to improve with a motility agent such as metoclopramide. Sucralfate, antacids, and selective antidepressants have not been shown to be more effective than placebo in functional dyspepsia, whereas tricyclic antidepressants and buspirone have shown some benefit and are reasonable next steps for this patient.

A 60-year-old male with COPD returns for a follow-up visit. He has been treated twice for pneumonia in the past year and is using his albuterol metered-dose inhaler (Proventil, Ventolin) several times a day. His FEV1 is 60% of predicted.To reduce symptoms and prevent hospitalization, which one of the following would be the most appropriate medication to add to this patient's treatment regimen at this time? (check one) A. Roflumilast (Daliresp) B. A long-acting β2-agonist bronchodilator C. A long-acting β2-agonist and an inhaled corticosteroid D. A long-acting anticholinergic inhaler E. Long-acting theophylline

To decrease the patient's frequent use of a short-acting bronchodilator, the addition of a long-acting anticholinergic inhaler for maintenance is most reasonable. Such medications have been demonstrated to improve quality of life and reduce hospitalizations. Evidence has demonstrated that a β-agonist combined with an inhaled corticosteroid would not be as preferable in this case, as they can increase the incidence of pneumonia. Long-acting theophylline and roflumilast may be added to the regimen of patients still symptomatic on triple therapy with long-acting β2-agonists, inhaled corticosteroids, and anticholinergic bronchodilators.

A 52-year-old female sees you because of a vaginal discharge. An examination reveals a malodorous, greenish-yellow, frothy discharge, and inflammation of the cervix and vagina. Which one of the following is the most likely diagnosis? (check one) A. Atrophic vaginitis B. Irritant/allergic vaginitis C. Bacterial vaginosis D. Trichomoniasis E. Vulvovaginal candidiasis

Trichomoniasis classically presents as a greenish-yellow, frothy discharge with a foul odor. Erythema and inflammation of the vagina and cervix are often present and can include punctate hemorrhages (strawberry cervix). Atrophic vaginitis may cause a thin, clear discharge and is usually associated with a thin, friable vaginal mucosa. Irritant/allergic vaginitis causes burning and soreness with vulvar erythema but usually does not cause any significant discharge. Bacterial vaginosis more commonly presents as a thin, homogenous discharge with a fishy odor and no cervical or vaginal inflammation. Vulvovaginal candidiasis presents with white, thick, cheesy, or curdy discharge.

A 36-year-old female calls your office because of a 2-day history of dysuria, urinary urgency, and urinary frequency. She has not had any fever, nausea, or vaginal discharge. She tells you her symptoms are similar to a previous urinary tract infection. She uses subdermal etonogestrel (Nexplanon) for contraception, takes no oral medications, and has no drug allergies. Which one of the following would be most appropriate at this point? (check one) A. Prescribe ciprofloxacin (Cipro) B. Prescribe nitrofurantoin (Macrobid, Macrodantin) C. Prescribe a urinary analgesic such as phenazopyridine (Pyridium) D. Ask the patient to come in today for evaluation E. Ask the patient to submit a urine specimen before you prescribe antibiotics

Urinary tract infection (UTI) is the most common bacterial infection in women. The annual incidence of UTI in women is 12%. Women who have had a UTI in the past are usually quite adept at diagnosing their own subsequent UTIs. Prospective studies have shown that women who suspect they have a UTI are more than 85% accurate based on culture results (SOR B). This is more accurate than dipstick testing, which has a sensitivity of 75% and a specificity of 82%. Nonpregnant female patients who have typical UTI symptoms without signs of pyelonephritis (i.e., fever and nausea) or vaginitis can be treated safely and effectively by phone. Urine culture testing is not indicated for uncomplicated UTIs. It has been found that the traditional criterion for infection (100,000 colony-forming units/mL) is not sensitive for women with a UTI. Urine testing should be reserved for patients suspected of having pyelonephritis. There are three first-line antibiotics for uncomplicated UTI. These include nitrofurantoin for 5 days, trimethoprim/sulfamethoxazole for 3 days, and fosfomycin as a single dose (SOR A). Fluoroquinolones are second-line agents and are best reserved for more serious infections such as pyelonephritis. Urinary analgesics can be helpful with UTI symptoms but are not the preferred method of treatment, as antibiotics rapidly reduce the symptoms of infection. Ref: Hooton TM: Uncomplicated urinary tract infection. N Engl J Med 2012;366(11):1028-1037. 2) Arnold JJ, Hehn LE, Klein DA: Common questions about recurrent urinary tract infections in women. Am Fam Physician 2016;93(7):560-569.

Absorption of levothyroxine is impaired by several gastrointestinal conditions, including atrophic gastritis, chronic proton pump inhibitor use, and Helicobacter pylori infection. Treatment of H. pylori infection reverses this effect, and following eradication of the infection a reduction of the levothyroxine dosage by 30% or more will often be required.

Which one of the following coexisting conditions could require the use of a much higher than expected dose of levothyroxine (Synthroid) to adequately treat hypothyroidism? (check one) A. Chronic kidney disease B. Diabetes mellitus C. Helicobacter pylori gastritis D. Hepatitis C infection E. Hyperparathyroidism

While you are examining a 3-day-old for a routine newborn checkup, his mother, who is also your patient, tells you that she has been crying very easily and feeling overwhelmed, and that some of her friends have suggested she ask about an antidepressant medication. She has no previous history of depression or other mood disorders. She does not have any thoughts of self-harm or intent to harm the baby or anyone else. She is breastfeeding and is otherwise healthy. Which one of the following would be most appropriate at this point? (check one) A. Reassurance that she likely has baby blues that will resolve in a few days B. Encouraging her to make an appointment with a therapist if she does not feel better in 1 week C. Referral to a counselor for psychotherapy D. Use of a validated screening tool to evaluate for postpartum depression E. Starting an SSRI for postpartum depression

While it is possible that this patient has baby blues, given the short duration of symptoms proximate to delivery, more evaluation is needed to differentiate between baby blues, which are very common and unlikely to contribute to the development of postpartum depression, and an episode of postpartum depression that has much more significant implications. There is more than one screening tool for postpartum depression, including the Edinburgh Postnatal Depression Scale, the PHQ-9 questionnaire, and the Postpartum Depression Screening Scale. An SSRI is the appropriate treatment for confirmed moderate to severe postpartum depression along with psychotherapy, while mild to moderate postpartum depression can be treated with psychotherapy or an SSRI. Referral to psychotherapy may be appropriate, but further evaluation should be pursued in order to determine the best treatment plan. Due to the risks associated with postpartum depression, an evaluation should be performed at this visit rather than asking the patient to monitor symptoms.

A 41-year-old male presents to the emergency department with severe left-sided back pain. He is afebrile and a urinalysis shows red blood cells. Imaging reveals a 6-mm stone in the lower ureter and no hydronephrosis.In addition to NSAIDs and narcotics for pain control, which one of the following would be most effective for hastening passage of the stone? (check one) A. An α1-blocker such as tamsulosin (Flomax) B. A 5-α-reductase inhibitor such as finasteride (Proscar) C. A nonselective β-blocker such as propranolol D. A phosphodiesterase inhibitor such as sildenafil (Viagra) E. A thiazide diuretic such as hydrochlorothiazide

α1-Blockers such as doxazosin, prazosin, and tamsulosin have been shown to hasten the passage of ureteral stones (level 2 evidence). They are probably more effective than calcium channel blockers (level 2 evidence). β-Blockers, phosphodiesterase inhibitors such as sildenafil, 5-α-reductase inhibitors such as finasteride, and thiazide diuretics have not been shown to hasten stone passage. However, thiazide diuretics have been shown to decrease stone formation in patients with hypercalciuria.


Related study sets

Social and Economic Effects of the Black Death

View Set

Chapter 33: all Disorders of Renal Function no explanation

View Set

Chapter 14, 15, 16, and 17 LEAP questions

View Set

Chapter 1: Patterns of Motion & Equilibrium

View Set

Auditing, Ch 7 Multiple choice questions

View Set